Суммарное сопротивление при параллельном соединении: Сопротивление при параллельном соединении, формула для расчета сопротивления при параллельном соединении

Содержание

Сопротивление при параллельном соединении, формула для расчета сопротивления при параллельном соединении

В этой статье мы разберем, как посчитать общее сопротивление при параллельном соединении сопротивлений. Параллельным соединением сопротивлений называется соединение (рисунок ниже), при котором один зажим каждого из сопротивлений присоединяется к одной точке (узлу) электрической цепи, а другой зажим каждого из тех же сопротивлений присоединяется к другой точке электрической цепи. Таким образом, между двумя точками (узлами) электрической цепи включается несколько сопротивлений, образующих параллельные ветви.

При этом напряжение между концами всех ветвей будет одним и тем же, а токи в отдельных ветвях определяются по закону Ома:
I1 = U / r1 ; I2 = U / r2 ; I3 = U / r3.

Напряжение U между узлами (А и Б):
U = I1r1 = I2r2 = I3r3,
откуда
I1 / I2 = R

2 / R1  и  I2 / I3 = R3 / R2,
т. е.

Токи в параллельных ветвях распределяются обратно пропорционально их сопротивлениям.

Согласно первому правилу Кирхгофа,
I = I1 + I2 + I3
или
U / Rсум = U / R1 + U / R2 + U / R2 = U (1 / R1 + 1 / R2 + 1 / R3).
Произведя сокращение на U, получим:
1 / Rсум = 1 / R1 + 1 / R2 + 1 / R3
или
g = g1 + g2 + g3 ,

где R и g—сопротивление и проводимость разветвленной цепи или, как их часто называют, общие сопротивление и проводимость цепи.
Из полученной формулы следует, что

Общая проводимость разветвленной цепи равна сумме проводимостей отдельных ветвей.

Формула
1 / Rсум = 1 / R1 + 1 / R2 + 1 / R3
дает возможность определить общее сопротивление цепи. Например, для трех параллельно соединенных сопротивлений, приведя правую часть уравнения к общему знаменателю, получим:
1 / Rсум = R2R3 + R1R3 + R1R2 / R1R

2R3
откуда
Rсум = R1R2R3 / R2R3 + R1R3 + R1R2
Если сопротивления R1 = R2 = R3, то общее сопротивление цепи:
Rсум = R1 / 3,
а в общем случае при n параллельных ветвях с равными сопротивлениями R1 :
Rсум = R1 / n
В случае двух параллельных ветвей:
1 / Rсум = 1 / R1 + 1 / R2
откуда
Rсум = R1R2
/ R2R3 + R1R3

При параллельном соединении приемников энергии все они находятся под одним и тем же напряжением, и режим работы каждого из них не зависит от остальных. Совершенно иначе обстоит дело при последовательном соединении приемников, при котором изменение сопротивления одного из них тотчас же приводит к изменению напряжения на других, последовательно соединенных с ним. Поэтому электрические лампы и двигатели, предназначенные для работы при определенном (номинальном) напряжении, включаются параллельно. Одинаковые электрические лампы иногда соединяются последовательно. Пусть, например, напряжение сети U, а напряжение лампы U

0 < U Тогда n ламп соединяются цепочкой друг за другом, причем n > U / U0
Такое соединение ламп можно встретить, например, в трамваях, метро и других случаях.

Пример 1:
К сети с напряжением 220 в параллельно подключены двигатель мощностью 1,1 квт и 11 ламп, каждая мощностью 40 вт. Определить ток в главных (подводящих) проводах

Ток двигателя
I1 = P1 / U = 1100 / 220 = 5a.
Ток ламп
I2 = P2 / U = 11 x 40 / 220 = 2a.
Ток в подводящих проводах
I = I1 + I2 = 5 + 2 = 7a.

Пример 2:
Определить общее сопротивление десяти параллельно включенных ламп накаливания, если каждая из них 240 ом:
R = R

л / n = 240 / 10 = 24ом.

формула и примеры расчета сопротивления, напряжения, тока и мощности

При проектировании электрических схем возникает необходимость использования последовательного и параллельного соединений резисторов. Соединения применяются также и при ремонтах электрооборудования, поскольку в некоторых ситуациях невозможно найти эквивалентный номинал резистора. Выполнить расчет просто, и справиться с этой операцией может каждый.

Типы проводников

Проводимость веществом электрического тока связана с наличием в нем свободных носителей заряда. Их количество определяется по электронной конфигурации. Для этого необходима химическая формула вещества, при помощи которой можно вычислить их общее число. Значение для каждого элемента берется из периодической системы Дмитрия Ивановича Менделеева.

Электрический ток — упорядоченное движение свободных носителей заряда, на которые воздействует электромагнитное поле. При протекании тока по веществу происходит взаимодействие потока заряженных частиц с узлами кристаллической решетки, при этом часть кинетической энергии частицы превращается в тепловую энергию. Иными словами, частица «ударяется» об атом, а затем снова продолжает движение, набирая скорость под действием электромагнитного поля.

Процесс взаимодействия частиц с узлами кристаллической решетки называется электрической проводимостью или сопротивлением материала. Единицей измерения является Ом, а определить его можно при помощи омметра или расчитать.

Согласно свойству проводимости, вещества можно разделить на 3 группы:

  1. Проводники (все металлы, ионизированный газ и электролитические растворы).
  2. Полупроводники (Si, Ge, GaAs, InP и InSb).
  3. Непроводники (диэлектрики или изоляторы).

Проводники всегда проводят электрический ток, поскольку содержат в своем атомарном строении свободные электроны, анионы, катионы и ионы. Полупроводники проводят электричество только при определенных условиях, которые влияют на наличие или отсутствие свободных электронов и дырок. К факторам, влияющим на проводимость, относятся следующие: температура, освещенность и т. д. Диэлектрики вообще не проводят электричество, поскольку в их структуре вообще отсутствуют свободные носители заряда. При выполнении расчетов каждый радиолюбитель должен знать зависимость сопротивления от некоторых физических величин.

Зависимость сопротивления

Значение электропроводимости зависит от нескольких факторов, которые необходимо учитывать при расчетах, изготовлении элементов резистивной нагрузки (резисторов), ремонте и проектировании устройств. К этим факторам необходимо отнести следующие:

  1. Температура окружающей среды и материала.
  2. Электрические величины.
  3. Геометрические свойства вещества.
  4. Тип материала, из которого изготовлен проводник (полупроводник).

К электрическим величинам можно отнести разность потенциалов (напряжение), электродвижущую силу (ЭДС) и силу тока. Геометрией проводника является его длина и площадь поперечного сечения.

Электрические величины

Зависимость величины электропроводимости от параметров электричества определяется законом Ома. Существует две формулировки: одна — для участка, а другая — для полной цепи. В первом случае соотношение определяются, исходя из значений силы тока (I) и напряжения (U) простой формулой: I = U / R. Из соотношения видна прямо пропорциональная зависимость тока от величины напряжения, а также обратно пропорциональная от сопротивления. Можно выразить R: R = U / I.

Для расчета электропроводимости всего участка следует воспользоваться соотношением между ЭДС (e), силой тока (i), а также внутренним сопротивлением источника питания (Rвн): i = e / (R+Rвн). В этом случае величина R вычисляется по формуле: R = (e / i) — Rвн. Однако при выполнении расчетов необходимо учитывать также геометрические параметры и тип проводника, поскольку они могут существенно повлиять на вычисления.

Тип и геометрические параметры

Свойство вещества к проводимости электричества определяется структурой кристаллической решетки, а также количеством свободных носителей. Исходя из этого, тип вещества является ключевым фактором, который определяет величину электропроводимости. В науке коэффициент, определяющий тип вещества, обозначается литерой «р» и называется удельным сопротивлением. Его значение для различных материалов (при температуре +20 градусов по Цельсию) можно найти в специальных таблицах.

Иногда для удобства расчетов используется обратная величина, которая называется удельной проводимостью (σ). Она связана с удельным сопротивлением следующим соотношением: p = 1 / σ. Площадь поперечного сечения (S) влияет на электрическое сопротивление. С физической точки зрения, зависимость можно понять следующим образом: при малом сечении происходят более частые взаимодействия частиц электрического тока с узлами кристаллической решетки.

Поперечное сечение можно вычислить по специальному алгоритму:

  1. Измерение геометрических параметров проводника (диаметр или длину сторон) при помощи штангенциркуля.
  2. Визуально определить форму материала.
  3. Вычислить площадь поперечного сечения по формуле, найденной в справочнике или интернете.

В случае когда проводник имеет сложную структуру, необходимо вычислить величину S одного элемента, а затем умножить результат на количество элементов, входящих в его состав. Например, если провод является многожильным, то следует вычислить S для одной жилы. После этого нужно умножить, полученную величину S, на количество жил. Зависимость R от вышеперечисленных величин можно записать в виде соотношения: R = p * L / S. Литера «L» является длиной проводника. Однако для получения точных расчетов необходимо учитывать температурные показатели внешней среды и проводника.

Температурные показатели

Существует доказательство зависимости удельного сопротивления материала от температуры, основанное на физическом эксперименте. Для проведения опыта нужно собрать электрическую цепь, состоящую из следующих элементов: источника питания, нихромовой спирали, соединительных проводов амперметра и вольтметра. Приборы нужны для измерения значений силы тока и напряжения соответственно. При протекании электричества происходит нагревание нихромовой пружины. По мере ее нагревания, показания амперметра уменьшаются. При этом происходит существенное падение напряжения на участке цепи, о котором свидетельствуют показания вольтметра.

В радиотехнике уменьшение величины напряжение называется просадкой или падением. Формула зависимости р от температуры имеет следующий вид: p = p0 * [1 + a * (t — 20)]. Значение p0 — удельное сопротивление материала, взятого из таблицы, а литера «t» — температура проводника.

Температурный коэффициент «а» принимает следующие значения: для металлов — a>0, а для электролитических растворов — a<0. Для получения формулы, определяющей все зависимости, необходимо подставить все соотношения в общую формулу зависимости R от типа материала, температуры, длины и сечения: R = p0 * [1 + a * (t — 20)] * L / S. Формулы используются только для расчетов и изготовления резисторов. Для быстрого измерения величины сопротивления применяется омметр.

Объединение резистивных радиокомпонентов

Для получения необходимого номинала сопротивления применяются два типа соединения резисторов: параллельное и последовательное. Если их соединить параллельно, то нужно два вывода одного резистора подключить к двум выводам другого. Если соединение является последовательным, то один вывод резистора соединяется с одним выводом другого резистора. Соединения используются для получения необходимых номиналов сопротивлений, а также для увеличения рассеивания мощности тока, протекающего по цепи.

Каждое из соединений обладает определенными характеристиками. Кроме того, последовательно или параллельно могут объединяться несколько резисторов. Соединения также могут быть смешанными, т. е. применяться оба типа объединения радиокомпонентов.

Параллельное соединение

При параллельном подключении значение напряжения на всех резисторах одинаковое, а сила тока — обратно пропорциональна их общему сопротивлению. В интернете web-разработчики создали для расчета величины общего сопротивления параллельного соединения резисторов онлайн-калькулятор.

Рассчитывается общее сопротивление при параллельном соединении по формуле: 1 / Rобщ = (1 / R1) + (1 / R2) + …+ (1 / Rn). Если выполнить математические преобразования и привести к общему знаменателю, то получится удобная формула параллельного соединения для расчета Rобщ. Она имеет следующий вид: Rобщ = (R1 * R2 * … * Rn) / (R1 + R2 + … + Rn). Если необходимо рассчитать величину Rобщ только для двух радиокомпонентов, то формула параллельного сопротивления имеет следующий вид: Rобщ = (R1 * R2) / (R1 + R2).

При ремонте или проектировании схемы устройства возникает задача объединения нескольких резистивных элементов для получения конкретной величины сопротивления. Например, значение Rобщ для определенной цепочки элементов равно 8 Ом, которое получено при расчетах. Перед радиолюбителем стоит задача, какие нужно подобрать номиналы для получения нужного значения (в стандартном ряду резисторов отсутствует радиокомпонент с номиналом в 8 Ом, а только 7,5 и 8,2). В этом случае нужно найти сопротивление при параллельном соединении резистивных элементов. Посчитать значение Rобщ для двух элементов можно следующим образом:

  1. Номинал резистора в 16 Ом подойдет.
  2. Подставить в формулу: R = (16 * 16) / (16 + 16) = 256 / 32 = 8 (Ом).

В некоторых случаях следует потратить больше времени на подбор необходимых номиналов. Можно применять не только два, но и три элемента. Сила тока вычисляется с использованием первого закона Кирхгофа. Формулировка закона следующая: общее значение тока, входящего и протекающего по цепи, равен выходному его значению. Величина силы тока для цепи, состоящей из двух резисторов (параллельное соединение) рассчитывается по такому алгоритму:

  1. Ток, протекающий через R1 и R2: I1 = U / R1 и I2 = U / R2 соответственно.
  2. Общий ток — сложение токов на резисторах: Iобщ = I1 + I2.

Например, если цепь состоит из 2 резисторов, соединенных параллельно, с номиналами в 16 и 7,5 Ом. Они запитаны от источника питания напряжением в 12 В. Значение силы тока на первом резисторе вычисляется следующим способом: I1 = 12 / 16 = 0,75 (А). На втором резисторе ток будет равен: I2 = 12 / 7,5 = 1,6 (А). Общий ток определяется по закону Кирхгофа: I = I1 + I2 = 1,6 + 0,75 = 2,35 (А).

Последовательное подключение

Последовательное включение резисторов также применяется в радиотехнике. Методы нахождения общего сопротивления, напряжения и тока отличаются от параллельного подключения. Основные правила соединения следующие:

  1. Ток не изменяется на участке цепи.
  2. Общее напряжение равно сумме падений напряжений на каждом резисторе.
  3. Rобщ = R1 + R2 + … + Rn.

Пример задачи следующий: цепочка, состоящая из 2 резисторов (16 и 7,5 Ом), питается от источника напряжением 12 В и током в 0,5 А. Необходимо рассчитать электрические параметры для каждого элемента. Порядок расчета следующий:

  1. I = I1 = I2 = 0,5 (А).
  2. Rобщ = R1 + R2 = 16 + 7,5 = 23,5 (Ом).
  3. Падения напряжения: U1 = I * R1 = 0,5 * 16 = 8 (В) и U2 = I * R2 = 0,5 * 7,5 = 3,75 (В).

Не всегда выполняется равенство напряжений (12 В не равно 8 + 3,75 = 11,75 В), поскольку при этом расчете не учитывается сопротивление соединительных проводов. Если схема является сложной, и в ней встречается два типа соединений, то нужно выполнять расчеты по участкам. В первую очередь, рассчитать для параллельного соединения, а затем для последовательного.

Таким образом, параллельное и последовательное соединения резисторов применяются для получения более точных значений сопротивлений, а также при отсутствии необходимого номинала радиокомпонента при проектировании или ремонте устройств.

Последовательное и параллельное соединение резисторов

Последовательное соединение резисторов

Последовательное соединениеэто соединение двух или более резисторов в форме цепи, в которой каждый отдельный резистор соединяется с другим отдельным резистором только в одной точке.

Общее сопротивление R

общ

При таком соединении, через все резисторы проходит один и тот же электрический ток. Чем больше элементов на данном участке электрической цепи, тем «труднее» току протекать через него. Следовательно, при последовательном соединении резисторов их общее сопротивление увеличивается, и оно равно сумме всех сопротивлений.

Напряжение при последовательном соединении

Напряжение при последовательном соединении распределяется на каждый резистор согласно закону Ома:

Т.е чем большее сопротивление резистора, тем большее напряжение на него падает.

Параллельное соединение резисторов

Параллельное соединениеэто соединение, при котором резисторы соединяются между собой обоими контактами. В результате к одной точке (электрическому узлу) может быть присоединено несколько резисторов.

Общее сопротивление R

общ

При таком соединении, через каждый резистор потечет отдельный ток. Сила данного тока будет обратно пропорциональна сопротивлению резистора. В результате общая проводимость такого участка электрической цепи увеличивается, а общее сопротивление в свою очередь уменьшается.

Таким образом, при параллельном подсоединении резисторов с разным сопротивлением, общее сопротивление будет всегда меньше значения самого маленького отдельного резистора.

Формула общей проводимости при параллельном соединении резисторов:

Формула эквивалентного общего сопротивления при параллельном соединении резисторов:

Для двух одинаковых резисторов общее сопротивление будет равно половине одного отдельного резистора:

Соответственно, для n одинаковых резисторов общее сопротивление будет равно значению одного резистора, разделенного на n.

Напряжение при параллельном соединении

Напряжение между точками A и B является как общим напряжением для всего участка цепи, так и напряжением, падающим на каждый резистор в отдельности. Поэтому при параллельном соединении на все резисторы упадет одинаковое напряжение.

Электрический ток при параллельном соединении

Через каждый резистор течет ток, сила которого обратно пропорциональна сопротивлению резистора. Для того чтобы узнать какой ток течет через определенный резистор, можно воспользоваться законом Ома:


Смешанное соединение резисторов

Смешанным соединением называют участок цепи, где часть резисторов соединяются между собой последовательно, а часть параллельно. В свою очередь, смешанное соединение бывает последовательного и параллельного типов.

Общее сопротивление R

общ

Для того чтобы посчитать общее сопротивление смешанного соединения:

  • Цепь разбивают на участки с только пареллельным или только последовательным соединением.
  • Вычисляют общее сопротивление для каждого отдельного участка.
  • Вычисляют общее сопротивление для всей цепи смешанного соединения.

Так это будет выглядеть для схемы 1:

Также существует более быстрый способ расчета общего сопротивления для смешанного соединения. Можно, в соответствии схеме, сразу записывать формулу следующим образом:

  • Если резисторы соединяются последоватеьно — складывать.
  • Если резисторы соединяются параллельно — использовать условное обозначение «||».
  • Подставлять формулу для параллельного соединения где стоит символ «||».

Так это будет выглядеть для схемы 1:

После подстановки формулы параллельного соединения вместо «||»:

Параллельное соединение проводников | Физика

При параллельном соединении все проводники (резисторы, лампы и т.д.) подключаются к одной и той же паре точек A и B (рис. 43). Связь между общими значениями силы тока, напряжения и сопротивления с их значениями на отдельных участках цепи при этом отличается от той, что была при последовательном соединении. Теперь соответствующие формулы имеют вид

I = I1 + I2, (17.1)     U = U1 = U2, (17.2)      R = (R1R2) / (R1 + R2). (17.3)

Чтобы убедиться в справедливости этих соотношений, следует собрать цепь и с помощью амперметра и вольтметра произвести необходимые измерения.

Итак, при параллельном соединении проводников напряжение на всех участках цепи одно и то же, общая сила тока равна сумме сил токов на отдельных проводниках, а общее сопротивление двух проводников находится как отношение произведения их сопротивлений к их сумме.

Первые две из этих закономерностей справедливы для любого числа параллельно соединенных проводников, последняя — только для двух.

Если R1 = R2, то

R = (R1R2) / (R1 + R2) = R12/2R1 = R1/2      (17.4)

Мы видим, что общее сопротивление двух одинаковых проводников в 2 раза меньше сопротивления одного проводника. Эта закономерность допускает обобщение: если параллельно соединено n одинаковых потребителей электроэнергии (резисторов, ламп и т.д.), то их общее сопротивление в n раз меньше сопротивления каждого из них:

R = R1/n      (17.5)

Отсюда следует, что с увеличением числа проводников общее сопротивление будет становиться все меньше и меньше. Это может показаться странным. На самом деле ничего удивительного в этом нет: ведь при параллельном соединении проводников происходит как бы увеличение общей площади их поперечного сечения, а с увеличением площади сечения проводника, как известно, его сопротивление уменьшается.

Отличительной особенностью параллельного соединения нескольких потребителей является то, что при выключении одного из них остальные продолжают работать. Так, например, вывернув одну лампу в цепи, изображенной на рисунке 44, мы увидим, что другая будет по-прежнему гореть.

Большинство потребителей электроэнергии — электронагревательные приборы, холодильники, швейные машины, магнитофоны, телевизоры и т. д. — рассчитаны на напряжение сети 220 В. Поэтому все они должны включаться в сеть параллельно, ибо только в этом случае они окажутся под одним и тем же напряжением (220 В) и будут продолжать работать при выключении одного из них.

На рисунке 45 приведена упрощенная схема квартирной электропроводки. Провода сети, между которыми существует напряжение 220 В, обозначены буквами Ф и О. Первый из них называют фазным, второй — нулевым. Нулевой провод соединен с землей. Именно с ним соединяют все потребители. И наоборот, все выключатели соединяют с фазным проводом. Такой порядок подключения потребителей и выключателей обеспечивает наибольшую безопасность человека.

??? 1. Какое соединение называют параллельным? 2. Начертите схему цепи, изображенной на рисунке 44. 3. Какие три закономерности справедливы для параллельного соединения проводников? 4. Как находится общее сопротивление параллельно соединенных проводников, когда они одинаковые? 5. Перечислите все элементы электрической цепи, изображенной на рисунке 45. 6. Предположим, что при замене лампы человек случайно коснулся металлического контакта в патроне лампы и одновременно с этим какой-либо заземленной части здания (например, батареи отопления). Под каким напряжением он окажется? Рассмотрите ситуацию, когда лампа и выключатель подсоединены к проводам сети так, как это показано на рисунке 45. Что произойдет, если лампу и выключатель поменять местами? 7. Почему у вольтметров делают большое внутреннее сопротивление, а у амперметров — малое?

Соединение резисторов — Основы электроники

Соединение резисторов в различные конфигурации очень часто применяются в электротехнике и электронике.
Здесь мы будем рассматривать только участок цепи, включающий в себя соединение резисторов.
Соединение резисторов может производиться последовательно, параллельно и смешанно (то есть и последовательно и параллельно), что показано на рисунке 1.

Рисунок 1. Соединение резисторов.

Последовательное соединение резисторов

Последовательное соединение резисторов это такое соединение, в котором конец одного резистора соединен с началом второго резистора, конец второго резистора с началом третьего и так далее (рисунок 2).

Рисунок 2. Последовательное соединение резисторов.

То есть при последовательном соединении резисторы подключатся друг за другом. При таком соединении через резисторы будет протекать один общий ток.
Следовательно, для последовательного соединения резисторов будет справедливо сказать, что между точками А и Б есть только один единственный путь протекания тока.
Таким образом, чем больше число последовательно соединенных резисторов, тем большее сопротивление они оказывают протеканию тока, то есть общее сопротивление Rобщ возрастает.
Рассчитывается общее сопротивление последовательно соединенных резисторов по следующей формуле:

Rобщ = R1 + R2 + R3+…+ Rn.

Параллельное соединение резисторов

Параллельное соединение резисторов это соединение, в котором начала всех резисторов соединены в одну общую точку (А), а концы в другую общую точку (Б) (см. рисунок 3).

Рисунок 3. Параллельное соединение резисторов.

При этом по каждому резистору течет свой ток. При параллельном соединении при протекании тока из точки А в точку Б, он имеет несколько путей.
Таким образом, увеличение числа параллельно соединенных резисторов ведет к увеличению путей протекания тока, то есть к уменьшению противодействия протеканию тока. А это значит, чем большее количество резисторов соединить параллельно, тем меньше станет значение общего сопротивления такого участка цепи (сопротивления между точкой А и Б.)
Общее сопротивление параллельно соединенных резисторов определяется следующим отношением:

1/Rобщ= 1/R1+1/R2+1/R3+…+1/Rn

Следует отметить, что здесь действует правило «меньше — меньшего». Это означает, что общее сопротивление всегда будет меньше сопротивления любого параллельно включенного резистора.
Общее сопротивление для двух параллельно соединенных резисторов рассчитывается по следующей формуле:

Rобщ= R1*R2/R1+R2

Если имеет место два параллельно соединенных резистора с одинаковыми сопротивлениями, то их общее сопротивление будет равно половине сопротивления одного из них.

Смешанное соединение резисторов

Смешанное соединение резисторов является комбинацией последовательного и параллельного соединения. Иногда подобную комбинацию называют последовательно-параллельным соединением.
На рисунке 4 показан простейший пример смешанного соединения резисторов.

Рисунок 4. Смешанное соединение резисторов.

На этом рисунке видно, что резисторы R2 R3 соединены параллельно, а R1, комбинация R2 R3 и R4 последовательно.
Для расчета сопротивления таких соединений, всю цепь разбивают на простейшие участки, из параллельно или последовательно соединенных резисторов. Далее следуют следующему алгоритму:
1. Определяют эквивалентное сопротивление участков с параллельным соединением резисторов.
2. Если эти участки содержат последовательно соединенные резисторы, то сначала вычисляют их сопротивление.
3. После расчета эквивалентных сопротивлений резисторов перерисовывают схему. Обычно получается цепь из последовательно соединенных эквивалентных сопротивлений.
4. Рассчитывают сопротивления полученной схемы.

Пример расчета участка цепи со смешанным соединением резисторов приведен на рисунке 5.

Рисунок 5. Расчет сопротивления участка цепи при смешанном соединении резисторов.

ПОНРАВИЛАСЬ СТАТЬЯ? ПОДЕЛИСЬ С ДРУЗЬЯМИ В СОЦИАЛЬНЫХ СЕТЯХ!

Похожие материалы:

Добавить комментарий

Последовательное и параллельное соединения проводников – FIZI4KA

1. Потребители электрической энергии: электрические лампочки, резисторы и пр. — могут по-разному соединяться друг с другом в электрической цепи. Существует два основных типа соединения проводников: последовательное и параллельное. При последовательном соединении проводников конец одного проводника соединяется с началом другого проводника, а его конец — с началом третьего и т.д. (рис. 85).

Примером последовательного соединения проводников может служить соединение электрических лампочек в ёлочной гирлянде.

При последовательном соединении проводников ток проходит через все лампочки, при этом через поперечное сечение каждого проводника в единицу времени проходит одинаковый заряд, т.е. заряд не скапливается ни в какой части проводника. Поэтому при последовательном соединении проводников сила тока в любом участке цепи одинакова: ​\( I_1=I_2=I \)​.

Общее сопротивление последовательно соединённых проводников равно сумме их сопротивлений: ​\( R_1=R_2=R \)​. Это следует из того, что при последовательном соединении проводников их общая длина увеличивается, она больше, чем длина каждого отдельного проводника, соответственно увеличивается и сопротивление проводников.

По закону Ома напряжение на каждом проводнике равно: ​\( U_1=IR_1 \)​, ​\( U_2=IR_2 \)​, а общее напряжение равно ​\( U=I(R_1+R_2) \)​. Поскольку сила тока во всех проводниках одинакова, а общее сопротивление равно сумме сопротивлений проводников, то полное напряжение на последовательно соединённых проводниках равно сумме напряжений на каждом проводнике: ​\( U=U_1+U_2 \)​.

Из приведённых равенств следует, что последовательное соединение проводников используется в том случае, если напряжение, на которое рассчитаны потребители электрической энергии, меньше общего напряжения в цепи.

2. Примером параллельного соединения проводников служит соединение потребителей электрической энергии в квартире. Так, электрические лампочки, чайник, утюг и пр. включаются параллельно.

При параллельном соединении проводников все проводники одним своим концом присоединяются к одной точке цепи (А), а вторым концом к другой точке цепи (В) (рис. 86).

Поэтому вольтметр, подключенный к этим точкам, покажет напряжение как на проводнике 1, так и на проводнике 2. Таким образом, напряжение на концах всех параллельно соединённых проводников одно и то же: ​\( U_1=U_2=U \)​.

При параллельном соединении проводников электрическая цепь разветвляется, в данном случае в точке В. Поэтому часть общего заряда проходит через один проводник, а часть — через другой. Следовательно при параллельном соединении проводников сила тока в неразветвлённой части цепи равна сумме силы тока в отдельных проводниках: ​\( I=I_1+I_2 \)​.

В соответствии с законом Ома ​\( I=\frac{U}{R} \)​, \( I_1=\frac{U_1}{R_1} \), \( I_2=\frac{U_2}{R_2} \). Отсюда следует: ​\( \frac{U}{R}=\frac{U_1}{R_1}+\frac{U_2}{R_2} \)​. Так как ​\( U_1=U_2=U \)​, \( \frac{1}{R}=\frac{1}{R_1}+\frac{1}{R_2} \). Величина, обратная общему сопротивлению параллельно соединенных проводников, равна сумме величин, обратных сопротивлению каждого проводника.

При параллельном соединении проводников их общее сопротивление меньше, чем сопротивление каждого проводника. Действительно, если параллельно соединены два проводника, имеющие одинаковое сопротивление ​\( r \)​, то их общее сопротивление равно: ​\( R=r/2 \)​. Это объясняется тем, что при параллельном соединении проводников как бы увеличивается площадь их поперечного сечения, соответственно уменьшается сопротивление.

Из приведённых формул понятно, почему потребители электрической энергии включаются параллельно: они все рассчитаны на определённое одинаковое напряжение, которое в квартирах равно 220 В. Зная сопротивление каждого потребителя, можно рассчитать силу тока в каждом из них и соответствие суммарной силы тока предельно допустимой силе тока.

ПРИМЕРЫ ЗАДАНИЙ

Часть 1

1. На рисунке изображёна схема участка электрической цепи АВ. В эту цепь параллельно включены два резистора сопротивлением ​\( R_1 \)​ и ​\( R_2 \)​. Напряжения на резисторах соответственно ​\( U_1 \)​ и ​\( U_2 \)​.

По какой из формул можно определить напряжение U на участке АВ?

1) ​\( U=U_1+U_2 \)​
2) ​\( U=U_1-U_2 \)​
3) ​\( U=U_1=U_2 \)​
4) ​\( U=\frac{U_1U_2}{U_1+U_2} \)​

2. На рисунке изображёна схема электрической цепи, содержащая два параллельно включённых резистора сопротивлением ​\( R_1 \)​ и ​\( R_2 \)​. Какое из приведённых ниже соотношений справедливо для такого соединения резисторов?

1) ​\( I=I_1=I_2 \)​
2) \( I=I_1+I_2 \)
3) \( U=U_1+U_2 \)
4) \( R=R_1+R_2 \)

3. На рисунке изображена схема электрической цепи. В эту цепь последовательно включены два резистора сопротивлением R} и R2. Какое из приведённых ниже соотношений справедливо для такого соединения резисторов?

1) ​\( U=U_1+U_2 \)​
2) \( I=I_1+I_2 \)
3) \( U=U_1=U_2 \)
4) \( R=\frac{R_1R_2}{R_1+R_2} \)

4. На рисунке изображена схема электрической цепи. В эту цепь последовательно включены два резистора сопротивлением ​\( R_1 \)​ и ​\( R_2 \)​. Какое из приведённых ниже соотношений справедливо для такого соединения резисторов?

1) ​\( U=U_1=U_2 \)​
2) \( I=I_1+I_2 \)
3) \( I=I_1=I_2 \)
4) \( R=\frac{R_1R_2}{R_1+R_2} \)

5.2 \)​
2) ​\( R=2R_1 \)​
3) ​\( R=\frac{R_1}{2} \)​
4) ​\( R=\sqrt{R_1} \)​

6. Общее сопротивление участка цепи, изображённого на рисунке, равно 9 Ом. Сопротивления резисторов ​\( R_1 \)​ и ​\( R_2 \)​ равны. Чему равно сопротивление каждого резистора?

1) 81 Ом
2) 18 Ом
3) 9 Ом
4) 4,5 Ом

 

7. Чему равно сопротивление участка цепи, содержащего три последовательно соединенных резистора сопротивлением по 9 Ом каждый?

1) 1/3 Ом
2) 3 Ом
3) 9 Ом
4) 27 Ом

8. Чему равно общее сопротивление участка цепи, изображённого на рисунке, если ​\( R_1 \)​ = 1 Ом, ​\( R_2 \)​ = 10 Ом, ​\( R_3 \)​ = 10 Ом, ​\( R_4 \)​ = 5 Ом?

1) 9 Ом
2) 11 Ом
3) 16 Ом
4) 26 Ом

9. Чему равно общее сопротивление участка цепи, изображённого на рисунке, если \( R_1 \) = 1 Ом, \( R_2 \) = 3 Ом, \( R_3 \) = 10 Ом, \( R_4 \) = 10 Ом?

1) 9 Ом
2) 10 Ом
3) 14 Ом
4) 24 Ом

10. Если ползунок реостата (см. схему) переместить влево, то сила тока

1) в резисторе ​\( R_1 \)​ уменьшится, а в резисторе ​\( R_2 \)​ увеличится
2) увеличится в обоих резисторах
3) в резисторе ​\( R_1 \)​ увеличится, а в резисторе ​\( R_2 \)​ уменьшится
4) уменьшится в обоих резисторах

11. На рисунке изображена электрическая цепь, состоящая из источника тока, резистора и реостата. Как изменяются при передвижении ползунка реостата вправо его сопротивление, сила тока в цепи и напряжение на резисторе 1?

Для каждой физической величины определите соответствующий характер изменения. Запишите в таблицу выбранные цифры для каждой физической величины. Цифры в ответе могут повторяться.

ФИЗИЧЕСКАЯ ВЕЛИЧИНА
A) сопротивление реостата 2
Б) сила тока в цепи
B) напряжение на резисторе 1

ХАРАКТЕР ИЗМЕНЕНИЯ
1) увеличивается
2) уменьшается
3) не изменяется

12. Установите соответствие между физическими величинами и правильной электрической схемой для измерения этих величин при последовательном соединении двух резисторов ​\( R_1 \)​ и \( R_2 \). Запишите в таблицу выбранные цифры под соответствующими буквами. Цифры в ответе могут повторяться.

ФИЗИЧЕСКИЕ ВЕЛИЧИНЫ
A) сила тока в резисторе \( R_1 \)​ и \( R_2 \)
Б) напряжение на резисторе \( R_2 \)
B) общее напряжение на резисторах \( R_1 \)​ и \( R_2 \)

Часть 2

13. Три резистора соединены, как показано на рисунке. Сопротивления резисторов ​\( R_1 \)​ = 10 Ом, \( R_2 \) = 5 Ом, \( R_3 \) = 5 Ом. Каково напряжение на резисторе 1, если амперметр показывает силу тока 2 А?

Ответы

Последовательное и параллельное соединения проводников

3.1 (62.86%) 7 votes

Суммарное сопротивление при последовательном соединении. Последовательное и параллельное соединение сопротивлений

Содержание:

Все известные виды проводников обладают определенными свойствами, в том числе и электрическим сопротивлением. Это качество нашло свое применение в резисторах, представляющих собой элементы цепи с точно установленным сопротивлением. Они позволяют выполнять регулировку тока и напряжения с высокой точностью в схемах. Все подобные сопротивления имеют свои индивидуальные качества. Например, мощность при паралл ельном и последовательном соединении резисторов будет различной. Поэтому на практике очень часто используются различные методики расчетов, благодаря которым возможно получение точных результатов.

Свойства и технические характеристики резисторов

Как уже отмечалось, резисторы в электрических цепях и схемах выполняют регулировочную функцию. С этой целью используется закон Ома, выраженный формулой: I = U/R. Таким образом, с уменьшением сопротивления происходит заметное возрастание тока. И, наоборот, чем выше сопротивление, тем меньше ток. Благодаря этому свойству, резисторы нашли широкое применение в электротехнике. На этой основе создаются делители тока, использующиеся в конструкциях электротехнических устройств.

Помимо функции регулировки тока, резисторы применяются в схемах делителей напряжения. В этом случае закон Ома будет выглядеть несколько иначе: U = I x R. Это означает, что с ростом сопротивления происходит увеличение напряжения. На этом принципе строится вся работа устройств, предназначенных для деления напряжения. Для делителей тока используется паралл ельное соединение резисторов, а для — последовательное.

На схемах резисторы отображаются в виде прямоугольника, размером 10х4 мм. Для обозначения применяется символ R, который может быть дополнен значением мощности данного элемента. При мощности свыше 2 Вт, обозначение выполняется с помощью римских цифр. Соответствующая надпись наносится на схеме возле значка резистора. Мощность также входит в состав , нанесенной на корпус элемента. Единицами измерения сопротивления служат ом (1 Ом), килоом (1000 Ом) и мегаом (1000000 Ом). Ассортимент резисторов находится в пределах от долей ома до нескольких сотен мегаом. Современные технологии позволяют изготавливать данные элементы с довольно точными значениями сопротивления.

Важным параметром резистора считается отклонение сопротивления. Его измерение осуществляется в процентах от номинала. Стандартный ряд отклонений представляет собой значения в виде: + 20, + 10, + 5, + 2, + 1% и так далее до величины + 0,001%.

Большое значение имеет мощность резистора. По каждому из них во время работы проходит электрический ток, вызывающий нагрев. Если допустимое значение рассеиваемой мощности превысит норму, это приведет к выходу из строя резистора. Следует учитывать, что в процессе нагревания происходит изменение сопротивления элемента. Поэтому если устройства работают в широких диапазонах температур, применяется специальная величина, именуемая температурным коэффициентом сопротивления.

Для соединения резисторов в схемах используются три разных способа подключения — паралл ельное, последовательное и смешанное. Каждый способ обладает индивидуальными качествами, что позволяет применять данные элементы в самых разных целях.

Мощность при последовательном соединение

При соединение резисторов последовательно электрический ток по очереди проходит через каждое сопротивление. Значение тока в любой точке цепи будет одинаковым. Данный факт определяется с помощью закона Ома. Если сложить все сопротивления, приведенные на схеме, то получится следующий результат: R = 200+100+51+39 = 390 Ом.

Учитывая напряжение в цепи, равное 100 В, сила тока будет составлять I = U/R = 100/390 = 0,256 A.На основании полученных данных можно рассчитать мощность резисторов при последовательном соединении по следующей формуле: P = I 2 x R = 0,256 2 x 390 = 25,55 Вт.

  • P 1 = I 2 x R 1 = 0,256 2 x 200 = 13,11 Вт;
  • P 2 = I 2 x R 2 = 0,256 2 x 100 = 6,55 Вт;
  • P 3 = I 2 x R 3 = 0,256 2 x 51 = 3,34 Вт;
  • P 4 = I 2 x R 4 = 0,256 2 x 39 = 2,55 Вт.

Если сложить полученные мощность, то полная Р составит: Р = 13,11+6,55+3,34+2,55 = 25,55 Вт.

Мощность при паралл ельном соединение

При паралл ельном подключении все начала резисторов соединяются с одним узлом схемы, а концы — с другим. В этом случае происходит разветвление тока, и он начинает протекать по каждому элементу. В соответствии с законом Ома, сила тока будет обратно пропорциональна всем подключенным сопротивлениям, а значение напряжения на всех резисторах будет одним и тем же.

Прежде чем вычислять силу тока, необходимо выполнить расчет полной проводимости всех резисторов, применяя следующую формулу:

  • 1/R = 1/R 1 +1/R 2 +1/R 3 +1/R 4 = 1/200+1/100+1/51+1/39 = 0,005+0,01+0,0196+0,0256 = 0,06024 1/Ом.
  • Поскольку сопротивление является величиной, обратно пропорциональной проводимости, его значение составит: R = 1/0,06024 = 16,6 Ом.
  • Используя значение напряжения в 100 В, по закону Ома рассчитывается сила тока: I = U/R = 100 x 0,06024 = 6,024 A.
  • Зная силу тока, мощность резисторов, соединенных паралл ельно, определяется следующим образом: P = I 2 x R = 6,024 2 x 16,6 = 602,3 Вт.
  • Расчет силы тока для каждого резистора выполняется по формулам: I 1 = U/R 1 = 100/200 = 0,5A; I 2 = U/R 2 = 100/100 = 1A; I 3 = U/R 3 = 100/51 = 1,96A; I 4 = U/R 4 = 100/39 = 2,56A. На примере этих сопротивлений прослеживается закономерность, что с уменьшением сопротивления, сила тока увеличивается.

Существует еще одна формула, позволяющая рассчитать мощность при паралл ельном подключении резисторов: P 1 = U 2 /R 1 = 100 2 /200 = 50 Вт; P 2 = U 2 /R 2 = 100 2 /100 = 100 Вт; P 3 = U 2 /R 3 = 100 2 /51 = 195,9 Вт; P 4 = U 2 /R 4 = 100 2 /39 = 256,4 Вт. Сложив мощности отдельных резисторов, получится их общая мощность: Р = Р 1 +Р 2 +Р 3 +Р 4 = 50+100+195,9+256,4 = 602,3 Вт.

Таким образом, мощность при последовательном и паралл ельном соединении резисторов определяется разными способами, с помощью которых можно получить максимально точные результаты.

Содержание:

Течение тока в электрической цепи осуществляется по проводникам, в направлении от источника к потребителям. В большинстве подобных схем используются медные провода и электрические приемники в заданном количестве, обладающие различным сопротивлением. В зависимости выполняемых задач, в электрических цепях используется последовательное и параллельное соединение проводников. В некоторых случаях могут быть применены оба типа соединений, тогда этот вариант будет называться смешанным. Каждая схема имеет свои особенности и отличия, поэтому их нужно обязательно заранее учитывать при проектировании цепей, ремонте и обслуживании электрооборудования.

Последовательное соединение проводников

В электротехнике большое значение имеет последовательное и параллельное соединение проводников в электрической цепи. Среди них часто используется схема последовательного соединения проводников предполагающая такое же соединение потребителей. В этом случае включение в цепь выполняется друг за другом в порядке очередности. То есть, начало одного потребителя соединяется с концом другого при помощи проводов, без каких-либо ответвлений.

Свойства такой электрической цепи можно рассмотреть на примере участков цепи с двумя нагрузками. Силу тока, напряжение и сопротивление на каждом из них следует обозначить соответственно, как I1, U1, R1 и I2, U2, R2. В результате, получились соотношения, выражающие зависимость между величинами следующим образом: I = I1 = I2, U = U1 + U2, R = R1 + R2. Полученные данные подтверждаются практическим путем с помощью проведения измерений амперметром и вольтметром соответствующих участков.

Таким образом, последовательное соединение проводников отличается следующими индивидуальными особенностями:

  • Сила тока на всех участках цепи будет одинаковой.
  • Общее напряжение цепи составляет сумму напряжений на каждом участке.
  • Общее сопротивление включает в себя сопротивления каждого отдельного проводника.

Данные соотношения подходят для любого количества проводников, соединенных последовательно. Значение общего сопротивления всегда выше, чем сопротивление любого отдельно взятого проводника. Это связано с увеличением их общей длины при последовательном соединении, что приводит и к росту сопротивления.

Если соединить последовательно одинаковые элементы в количестве n, то получится R = n х R1, где R — общее сопротивление, R1 — сопротивление одного элемента, а n — количество элементов. Напряжение U, наоборот, делится на равные части, каждая из которых в n раз меньше общего значения. Например, если в сеть с напряжением 220 вольт последовательно включаются 10 ламп одинаковой мощности, то напряжение в любой из них составит: U1 = U/10 = 22 вольта.

Проводники, соединенные последовательно, имеют характерную отличительную особенность. Если во время работы отказал хотя-бы один из них, то течение тока прекращается во всей цепи. Наиболее ярким примером является , когда одна перегоревшая лампочка в последовательной цепи, приводит к выходу из строя всей системы. Для установления перегоревшей лампочки понадобится проверка всей гирлянды.

Параллельное соединение проводников

В электрических сетях проводники могут соединяться различными способами: последовательно, параллельно и комбинированно. Среди них параллельное соединение это такой вариант, когда проводники в начальных и конечных точках соединяются между собой. Таким образом, начала и концы нагрузок соединяются вместе, а сами нагрузки располагаются параллельно относительно друг друга. В электрической цепи могут содержаться два, три и более проводников, соединенных параллельно.

Если рассматривать последовательное и параллельное соединение, сила тока в последнем варианте может быть исследована с помощью следующей схемы. Берутся две лампы накаливания, обладающие одинаковым сопротивлением и соединенные параллельно. Для контроля к каждой лампочке подключается собственный . Кроме того, используется еще один амперметр, контролирующий общую силу тока в цепи. Проверочная схема дополняется источником питания и ключом.

После замыкания ключа нужно контролировать показания измерительных приборов. Амперметр на лампе № 1 покажет силу тока I1, а на лампе № 2 — силу тока I2. Общий амперметр показывает значение силы тока, равное сумме токов отдельно взятых, параллельно соединенных цепей: I = I1 + I2. В отличие от последовательного соединения, при перегорании одной из лампочек, другая будет нормально функционировать. Поэтому в домашних электрических сетях используется параллельное подключение приборов.

С помощью такой же схемы можно установить значение эквивалентного сопротивления. С этой целью в электрическую цепь добавляется вольтметр. Это позволяет измерить напряжение при параллельном соединении, сила тока при этом остается такой же. Здесь также имеются точки пересечения проводников, соединяющих обе лампы.

В результате измерений общее напряжение при параллельном соединении составит: U = U1 = U2. После этого можно рассчитать эквивалентное сопротивление, условно заменяющее все элементы, находящиеся в данной цепи. При параллельном соединении, в соответствии с законом Ома I = U/R, получается следующая формула: U/R = U1/R1 + U2/R2, в которой R является эквивалентным сопротивлением, R1 и R2 — сопротивления обеих лампочек, U = U1 = U2 — значение напряжения, показываемое вольтметром.

Следует учитывать и тот фактор, что токи в каждой цепи, в сумме составляют общую силу тока всей цепи. В окончательном виде формула, отражающая эквивалентное сопротивление будет выглядеть следующим образом: 1/R = 1/R1 + 1/R2. При увеличении количества элементов в таких цепях — увеличивается и число слагаемых в формуле. Различие в основных параметрах отличают друг от друга и источников тока, позволяя использовать их в различных электрических схемах.

Параллельное соединение проводников характеризуется достаточно малым значением эквивалентного сопротивления, поэтому сила тока будет сравнительно высокой. Данный фактор следует учитывать, когда в розетки включается большое количество электроприборов. В этом случае сила тока значительно возрастает, приводя к перегреву кабельных линий и последующим возгораниям.

Законы последовательного и параллельного соединения проводников

Данные законы, касающиеся обоих видов соединений проводников, частично уже были рассмотрены ранее.

Для более четкого их понимания и восприятия в практической плоскости, последовательное и параллельное соединение проводников, формулы следует рассматривать в определенной последовательности:

  • Последовательное соединение предполагает одинаковую силу тока в каждом проводнике: I = I1 = I2.
  • параллельное и последовательное соединение проводников объясняет в каждом случае по-своему. Например, при последовательном соединении, напряжения на всех проводниках будут равны между собой: U1 = IR1, U2 = IR2. Кроме того, при последовательном соединении напряжение составляет сумму напряжений каждого проводника: U = U1 + U2 = I(R1 + R2) = IR.
  • Полное сопротивление цепи при последовательном соединении состоит из суммы сопротивлений всех отдельно взятых проводников, независимо от их количества.
  • При параллельном соединении напряжение всей цепи равно напряжению на каждом из проводников: U1 = U2 = U.
  • Общая сила тока, измеренная во всей цепи, равна сумме токов, протекающих по всем проводникам, соединенных параллельно между собой: I = I1 + I2.

Для того чтобы более эффективно проектировать электрические сети, нужно хорошо знать последовательное и параллельное соединение проводников и его законы, находя им наиболее рациональное практическое применение.

Смешанное соединение проводников

В электрических сетях как правило используется последовательное параллельное и смешанное соединение проводников, предназначенное для конкретных условий эксплуатации. Однако чаще всего предпочтение отдается третьему варианту, представляющему собой совокупность комбинаций, состоящих из различных типов соединений.

В таких смешанных схемах активно применяется последовательное и параллельное соединение проводников, плюсы и минусы которых обязательно учитываются при проектировании электрических сетей. Эти соединения состоят не только из отдельно взятых резисторов, но и довольно сложных участков, включающих в себя множество элементов.

Смешанное соединение рассчитывается в соответствии с известными свойствами последовательного и параллельного соединения. Метод расчета заключается в разбивке схемы на более простые составные части, которые считаются отдельно, а потом суммируются друг с другом.

Параллельное соединение резисторов. При параллельном соединении резисторов нескольких приемников они включаются между двумя точками электрической цепи, образуя параллельные ветви (рис. 26, а). Заменяя

лампы резисторами с сопротивлениями R1, R2, R3, получим схему, показанную на рис. 26, б.
При параллельном соединении ко всем резисторам приложено одинаковое напряжение U. Поэтому согласно закону Ома:

I 1 =U/R 1 ; I 2 =U/R 2 ; I 3 =U/R 3 .

Ток в неразветвленной части цепи согласно первому закону Кирхгофа I = I 1 +I 2 +I 3 , или

I = U / R 1 + U / R 2 + U / R 3 = U (1/R 1 + 1/R 2 + 1/R 3) = U / R эк (23)

Следовательно, эквивалентное сопротивление рассматриваемой цепи при параллельном соединении трех резисторов определяется формулой

1/R эк = 1/R 1 + 1/R 2 + 1/R 3 (24)

Вводя в формулу (24) вместо значений 1/R эк, 1/R 1 , 1/R 2 и 1/R 3 соответствующие проводимости G эк, G 1 , G 2 и G 3 , получим: эквивалентная проводимость параллельной цепи равна сумме проводимостей параллельно соединенных резисторов :

G эк = G 1 + G 2 +G 3 (25)

Таким образом, при увеличении числа параллельно включаемых резисторов результирующая проводимость электрической цепи увеличивается, а результирующее сопротивление уменьшается.
Из приведенных формул следует, что токи распределяются между параллельными ветвями обратно пропорционально их электрическим сопротивлениям или прямо пропорционально их проводимостям. Например, при трех ветвях

I 1: I 2: I 3 = 1/R 1: 1/R 2: 1/R 3 = G 1 + G 2 + G 3 (26)

В этом отношении имеет место полная аналогия между распределением токов по отдельным ветвям и распределением потоков воды по трубам.
Приведенные формулы дают возможность определить эквивалентное сопротивление цепи для различных конкретных случаев. Например, при двух параллельно включенных резисторах результирующее сопротивление цепи

R эк =R 1 R 2 /(R 1 +R 2)

при трех параллельно включенных резисторах

R эк =R 1 R 2 R 3 /(R 1 R 2 +R 2 R 3 +R 1 R 3)

При параллельном соединении нескольких, например n, резисторов с одинаковым сопротивлением R1 результирующее сопротивление цепи Rэк будет в n раз меньше сопротивления R1, т.е.

R эк = R1 / n (27)

Проходящий по каждой ветви ток I1, в этом случае будет в п раз меньше общего тока:

I1 = I / n (28)

При параллельном соединении приемников, все они находятся под одним и тем же напряжением, и режим работы каждого из них не зависит от остальных. Это означает, что ток, проходящий по какому-либо из приемников, не будет оказывать существенного влияния на другие приемники. При всяком выключении или выходе из строя любого приемника остальные приемники остаются включенными. Поэтому параллельное соединение имеет существенные преимущества перед последовательным, вследствие чего оно получило наиболее широкое распространение. В частности, электрические лампы и двигатели, предназначенные для работы при определенном (номинальном) напряжении, всегда включают параллельно.
На электровозах постоянного тока и некоторых тепловозах тяговые двигатели в процессе регулирования скорости движения нужно включать под различные напряжения, поэтому они в процессе разгона переключаются с последовательного соединения на параллельное.

Каждый в этой жизни сталкивался с резисторами. Люди с гуманитарными профессиями, как и все, изучали в школе на уроках физики проводники электрического тока и закон Ома.

С резисторами также имеют дело студенты технических университетов и инженеры различных производственных предприятий. Перед всеми этими людьми, так или иначе, вставала задача расчёта электрической цепи при различных видах соединения резисторов. В данной статье речь пойдёт о расчёте физических параметров, характеризующих цепь.

Виды соединений

Резистор — пассивный элемент , присутствующий в каждой электрической цепи. Он предназначен для того, чтобы сопротивляться электрическому току. Существует два вида резисторов:

  1. Постоянные.
  2. Переменные.

Зачем же спаивать проводники друг с другом? Например, если для какой-то электрической цепи нужно определённое сопротивление. А среди номинальных показателей нужного нет. В таком случае необходимо подобрать элементы схемы с определёнными значениями сопротивления и соединить их. В зависимости от вида соединения и сопротивлений пассивных элементов мы получим какое-то определённое сопротивление цепи. Оно называется эквивалентным. Его значение зависит от вида спайки проводников. Существует три вида соединения проводников:

  1. Последовательное.
  2. Параллельное.
  3. Смешанное.

Значение эквивалентного сопротивления в цепи считается достаточно легко. Однако, если резисторов в схеме очень много, то лучше воспользоваться специальным калькулятором, который считает это значение. При ведении расчёта вручную, чтобы не допускать ошибок, необходимо проверять, ту ли формулу вы взяли.

Последовательное соединение проводников

В последовательной спайке резисторы идут как бы друг за другом. Значение эквивалентного сопротивления цепи равно сумме сопротивлений всех резисторов. Особенность схем с такой спайкой заключается в том, что значение тока постоянно . Согласно закону Ома, напряжение в цепи равно произведению тока и сопротивления. Так как ток постоянен, то для вычисления напряжения на каждом резисторе, достаточно перемножить значения. После этого необходимо сложить напряжения всех резисторов, и тогда мы получим значение напряжения во всей цепи.

Расчёт очень простой. Так как с ним имеют дело в основном инженеры-разработчики, то для них не составит труда сосчитать всё вручную. Но если резисторов очень много, то проще воспользоваться специальным калькулятором.

Примером последовательного соединения проводников в быту является ёлочная гирлянда.

Параллельное соединение резисторов

При параллельном соединении проводников эквивалентное сопротивление в цепи считается по-другому. Немного сложнее, чем при последовательном.

Его значение в таких цепях равняется произведению сопротивлений всех резисторов, делённому на их сумму. А также есть и другие варианты этой формулы. Параллельное соединение резисторов всегда снижает эквивалентное сопротивление цепи. То есть, его значение всегда будет меньше, чем наибольшее значение какого-то из проводников.

В таких схемах значение напряжения постоянно . То есть значение напряжения во всей цепи равно значениям напряжений каждого из проводников. Оно задаётся источником напряжения.

Сила тока в цепи равна сумме всех токов, протекающих через все проводники. Значение силы тока, протекающего через проводник. равно отношению напряжения источника к сопротивлению этого проводника.

Примеры параллельного соединения проводников:

  1. Освещение.
  2. Розетки в квартире.
  3. Производственное оборудование.

Для расчёта схем с параллельным соединением проводников лучше пользоваться специальным калькулятором. Если в схеме много резисторов, спаянных параллельно, то гораздо быстрее вы посчитаете эквивалентное сопротивление с помощью этого калькулятора.

Смешанное соединение проводников

Этот вид соединения состоит из каскадов резисторов . Например, у нас есть каскад из 10 проводников, соединённых последовательно, и после него идёт каскад из 10 проводников, соединённых параллельно. Эквивалентное сопротивление этой схемы будет равно сумме эквивалентных сопротивлений этих каскадов. То есть, по сути, здесь последовательное соединение двух каскадов проводников.

Многие инженеры занимаются оптимизацией различных схем. Её целью является уменьшение количества элементов в схеме за счёт подбора других, с подходящими значениями сопротивлений. Сложные схемы разбиваются на несколько небольших каскадов, ведь так гораздо проще вести расчёты.

Сейчас, в двадцать первом веке, инженерам стало гораздо проще работать. Ведь несколько десятилетий назад все расчёты производились вручную. А сейчас программисты разработали специальный калькулятор для расчёта эквивалентного сопротивления цепи. В нём запрограммированы формулы, по которым ведутся расчёты.

В этом калькуляторе можно выбрать вид соединения, и потом ввести в специальные поля значения сопротивлений. Через несколько секунд вы уже увидите это значение.

Элементы электрической цепи можно соединить двумя способами. Последовательное соединение подразумевает подключение элементов друг к другу, а при параллельном соединении элементы являются частью параллельных ветвей. Способ соединения резисторов определяет метод вычисления общего сопротивления цепи.

Шаги

Последовательное соединение

    Определите, является ли цепь последовательной. Последовательное соединение представляет собой единую цепь без каких-либо разветвлений. Резисторы или другие элементы расположены друг за другом.

    Сложите сопротивления отдельных элементов. Сопротивление последовательной цепи равно сумме сопротивлений всех элементов, входящих в эту цепь. Сила тока в любых частях последовательной цепи одна и та же, поэтому сопротивления просто складываются.

  • Например, последовательная цепь состоит из трех резисторов с сопротивлениями 2 Ом, 5 Ом и 7 Ом. Общее сопротивление цепи: 2 + 5 + 7 = 14 Ом.
  • Если сопротивление каждого элемента цепи не известно, воспользуйтесь законом Ома: V = IR, где V – напряжение, I – сила тока, R – сопротивление. Сначала найдите силу тока и общее напряжение.

    Подставьте известные значения в формулу, описывающую закон Ома. Перепишите формулу V = IR так, чтобы обособить сопротивление: R = V/I. Подставьте известные значения в эту формулу, чтобы вычислить общее сопротивление.

    • Например, напряжение источника тока равно 12 В, а сила тока равна 8 А. Общее сопротивление последовательной цепи: R O = 12 В / 8 А = 1,5 Ом.
  • Параллельное соединение

    1. Определите, является ли цепь параллельной. Параллельная цепь на некотором участке разветвляется на несколько ветвей, которые затем снова соединяются. Ток течет по каждой ветви цепи.

      Вычислите общее сопротивление на основе сопротивления каждой ветви. Каждый резистор уменьшает силу тока, проходящего через одну ветвь, поэтому она оказывает небольшое влияние на общее сопротивление цепи. Формула для вычисления общего сопротивления: , где R 1 – сопротивление первой ветви, R 2 – сопротивление второй ветви и так далее до последней ветви R n .

      Вычислите сопротивление по известной силе тока и напряжению. Сделайте это, если сопротивление каждого элемента цепи не известно.

      Подставьте известные значения в формулу закона Ома. Если известны значения общей силы тока и напряжения в цепи, общее сопротивление вычисляется по закону Ома: R = V/I.

      • Например, напряжение в параллельной цепи равно 9 В, а общая сила тока равна 3 А. Общее сопротивление: R O = 9 В / 3 А = 3 Ом.
    2. Поищите ветви с нулевым сопротивлением. Если у ветви параллельной цепи вообще нет сопротивления, то весь ток будет течь через такую ветвь. В этом случае общее сопротивление цепи равно 0 Ом.

    Комбинированное соединение

      Разбейте комбинированную цепь на последовательную и параллельную. Комбинированная цепь включает элементы, которые соединены как последовательно, так и параллельно. Посмотрите на схему цепи и подумайте, как разбить ее на участки с последовательным и параллельным соединением элементов. Обведите каждый участок, чтобы упростить задачу по вычислению общего сопротивления.

    • Например, цепь включает резистор, сопротивление которого равно 1 Ом, и резистор, сопротивление которого равно 1,5 Ом. За вторым резистором схема разветвляется на две параллельные ветви – одна ветвь включает резистор с сопротивлением 5 Ом, а вторая – с сопротивлением 3 Ом. Обведите две параллельные ветви, чтобы выделить их на схеме цепи.
  • Найдите сопротивление параллельной цепи. Для этого воспользуйтесь формулой для вычисления общего сопротивления параллельной цепи: 1 R O = 1 R 1 + 1 R 2 + 1 R 3 + . . . 1 R n {\displaystyle {\frac {1}{R_{O}}}={\frac {1}{R_{1}}}+{\frac {1}{R_{2}}}+{\frac {1}{R_{3}}}+…{\frac {1}{R_{n}}}} .

    Упростите цепь. После того как вы нашли общее сопротивление параллельной цепи, ее можно заменить одним элементом, сопротивление которого равно вычисленному значению.

    • В нашем примере избавьтесь от двух параллельных ветвей и замените их одним резистором с сопротивлением 1,875 Ом.
  • Сложите сопротивления резисторов, соединенных последовательно. Заменив параллельную цепь одним элементом, вы получили последовательную цепь. Общее сопротивление последовательной цепи равно сумме сопротивлений всех элементов, которые включены в эту цепь.

  • резисторов последовательно и параллельно

    Резисторы серии

    Общее сопротивление в цепи с последовательно включенными резисторами равно сумме отдельных сопротивлений.

    Цели обучения

    Рассчитайте общее сопротивление в цепи с последовательно включенными резисторами

    Основные выводы

    Ключевые моменты
    • Одинаковый ток течет через каждый резистор последовательно.
    • Отдельные последовательно включенные резисторы не получают полное напряжение источника, а делят его.
    • Общее сопротивление в последовательной цепи равно сумме отдельных сопротивлений: [латекс] \ text {RN} (\ text {series}) = \ text {R} _1 + \ text {R} _2 + \ text {R} _3 +… + \ text {R} _ \ text {N} [/ latex].
    Ключевые термины
    • серия : ряд элементов, которые следуют одно за другим или связаны друг за другом.
    • сопротивление : Противодействие прохождению электрического тока через этот элемент.

    Обзор

    Большинство схем имеет более одного компонента, называемого резистором, который ограничивает поток заряда в цепи.Мера этого предела для потока заряда называется сопротивлением. Самыми простыми комбинациями резисторов являются последовательное и параллельное соединение. Общее сопротивление комбинации резисторов зависит как от их индивидуальных значений, так и от способа их подключения.

    Цепи серии : Краткое введение в анализ последовательных и последовательных цепей, включая закон Кирхгофа по току (KCL) и закон Кирхгофа по напряжению (KVL).

    Резисторы серии

    Резисторы включены последовательно, когда заряд или ток должны проходить через компоненты последовательно.

    Резисторы в серии : Эти четыре резистора подключены последовательно, потому что, если бы ток подавался на один конец, он бы протекал через каждый резистор последовательно до конца.

    показывает резисторы, последовательно подключенные к источнику напряжения. Общее сопротивление в цепи равно сумме отдельных сопротивлений, поскольку ток должен проходить через каждый резистор последовательно через цепь.

    Резисторы, подключенные последовательно. : Три резистора, подключенные последовательно к батарее (слева), и эквивалентное одиночное или последовательное сопротивление (справа).

    Использование закона Ома для расчета изменений напряжения в резисторах серии

    Согласно закону Ома падение напряжения V на резисторе при протекании через него тока рассчитывается по формуле V = IR, где I — ток в амперах (A), а R — сопротивление в омах (Ω). .

    Таким образом, падение напряжения на R 1 составляет В 1 = IR 1 , на R 2 составляет В 2 = IR 2 , а на R 3 составляет В 3 = IR 3 .Сумма напряжений будет равна: V = V 1 + V 2 + V 3 , исходя из сохранения энергии и заряда. Если подставить значения отдельных напряжений, получим:

    [латекс] \ text {V} = \ text {IR} _1 + \ text {IR} _2 + \ text {IR} _3 [/ latex]

    или

    [латекс] \ text {V} = \ text {I} (\ text {R} _1 + \ text {R} _2 + \ text {R} _3) [/ латекс]

    Это означает, что полное сопротивление в серии равно сумме отдельных сопротивлений. Следовательно, для каждой цепи с Н количество резисторов, включенных последовательно:

    [латекс] \ text {RN} (\ text {series}) = \ text {R} _1 + \ text {R} _2 + \ text {R} _3 +… + \ text {R} _ \ text {N }.[/ латекс]

    Поскольку весь ток должен проходить через каждый резистор, он испытывает сопротивление каждого, а последовательно соединенные сопротивления просто складываются.

    Поскольку напряжение и сопротивление имеют обратную зависимость, отдельные последовательно включенные резисторы не получают полное напряжение источника, а делят его. Об этом свидетельствует пример, когда две лампочки соединены в последовательную цепь с аккумулятором. В простой схеме, состоящей из одной батареи 1,5 В и одной лампочки, падение напряжения на лампе будет равно 1.5V через него. Однако, если бы две лампочки были соединены последовательно с одной и той же батареей, на каждой из них было бы падение напряжения 1,5 В / 2 или 0,75 В. Это будет очевидно по яркости света: каждая из двух последовательно соединенных лампочек будет в два раза слабее, чем одиночная лампочка. Следовательно, резисторы, соединенные последовательно, потребляют такое же количество энергии, как и один резистор, но эта энергия распределяется между резисторами в зависимости от их сопротивлений.

    Параллельные резисторы

    Общее сопротивление в параллельной цепи равно сумме обратных сопротивлений каждого отдельного сопротивления.

    Цели обучения

    Рассчитайте общее сопротивление в цепи с параллельно включенными резисторами

    Основные выводы

    Ключевые моменты
    • Общее сопротивление в параллельной цепи меньше наименьшего из отдельных сопротивлений.
    • Каждый резистор, включенный параллельно, имеет то же напряжение, что и приложенный к нему источник (напряжение в параллельной цепи постоянно).
    • Не каждый параллельный резистор получает полный ток; они делят его (ток зависит от номинала каждого резистора и общего количества резисторов в цепи).
    Ключевые термины
    • сопротивление : Противодействие прохождению электрического тока через этот элемент.
    • параллельно : Расположение электрических компонентов, при котором ток течет по двум или более путям.

    Обзор

    Резисторы в цепи могут быть включены последовательно или параллельно. Общее сопротивление комбинации резисторов зависит как от их индивидуальных значений, так и от способа их подключения.

    Параллельные схемы : Краткий обзор анализа параллельных цепей с использованием таблиц VIRP для студентов-физиков средней школы.

    Параллельные резисторы

    Резисторы включены параллельно, когда каждый резистор подключен непосредственно к источнику напряжения путем соединения проводов с незначительным сопротивлением. Таким образом, к каждому резистору приложено полное напряжение источника.

    Параллельное соединение резисторов : Параллельное соединение резисторов.

    Каждый резистор потребляет такой же ток, как если бы он был единственным резистором, подключенным к источнику напряжения. Это верно для схем в доме или квартире. Каждая розетка, подключенная к устройству («резистор»), может работать независимо, и ток не должен проходить через каждое устройство последовательно.

    Закон

    Ом и параллельные резисторы

    Каждый резистор в цепи имеет полное напряжение. Согласно закону Ома токи, протекающие через отдельные резисторы, равны [латекс] \ text {I} _1 = \ frac {\ text {V}} {\ text {R} _1} [/ latex], [latex] \ text {I} _2 = \ frac {\ text {V}} {\ text {R} _2} [/ latex] и [latex] \ text {I} _3 = \ frac {\ text {V}} {\ text {R} _3} [/ латекс].Сохранение заряда подразумевает, что полный ток равен сумме этих токов:

    Параллельные резисторы : три резистора, подключенные параллельно батарее, и эквивалентное одиночное или параллельное сопротивление.

    [латекс] \ text {I} = \ text {I} _1 + \ text {I} _2 + \ text {I} _3. [/ Latex]

    Подстановка выражений для отдельных токов дает:

    [латекс] \ text {I} = \ frac {\ text {V}} {\ text {R} _1} + \ frac {\ text {V}} {\ text {R} _2} + \ frac {\ текст {V}} {\ text {R} _3} [/ latex]

    или

    [латекс] \ text {I} = \ text {V} (\ frac {1} {\ text {R} _1} + \ frac {1} {\ text {R} _2} + \ frac {1} { \ text {R} _3}) [/ latex]

    Это означает, что полное сопротивление в параллельной цепи равно сумме обратных сопротивлений каждого отдельного сопротивления.Следовательно, для каждой схемы с числом [latex] \ text {n} [/ latex] или параллельно подключенных резисторов —

    [латекс] \ text {R} _ {\ text {n} \; (\ text {parallel})} = \ frac {1} {\ text {R} _1} + \ frac {1} {\ text { R} _2} + \ frac {1} {\ text {R} _3}… + \ frac {1} {\ text {R} _ \ text {n}}. [/ Latex]

    Это соотношение приводит к общему сопротивлению, которое меньше наименьшего из отдельных сопротивлений. Когда резисторы соединены параллельно, от источника течет больше тока, чем протекает для любого из них по отдельности, поэтому общее сопротивление ниже.

    Каждый резистор, включенный параллельно, имеет такое же полное напряжение источника, как на него, но делит общий ток между ними. Примером может служить соединение двух лампочек в параллельную цепь с аккумулятором на 1,5 В. В последовательной цепи две лампочки будут вдвое менее тусклыми при подключении к одному источнику батареи. Однако, если бы две лампочки были подключены параллельно, они были бы столь же яркими, как если бы они были подключены к батарее по отдельности. Поскольку к обеим лампочкам подается одинаковое полное напряжение, батарея также разряжается быстрее, поскольку она по существу обеспечивает полную энергию обеими лампочками.В последовательной цепи батарея будет работать столько же, сколько и с одной лампочкой, только тогда яркость будет разделена между лампочками.

    Комбинированные схемы

    Комбинированная схема может быть разбита на аналогичные части, которые работают последовательно или параллельно.

    Цели обучения

    Описать расположение резисторов в комбинированной цепи и его практическое значение

    Основные выводы

    Ключевые моменты
    • Более сложные соединения резисторов иногда представляют собой просто комбинации последовательного и параллельного.
    • Различные части комбинированной схемы могут быть идентифицированы как последовательные или параллельные, уменьшены до их эквивалентов, а затем уменьшены до тех пор, пока не останется единственное сопротивление.
    • Сопротивление в проводах снижает ток и мощность, подаваемые на резистор. Если сопротивление в проводах относительно велико, как в изношенном (или очень длинном) удлинительном шнуре, то эти потери могут быть значительными и влиять на выходную мощность в устройствах.
    Ключевые термины
    • серия : ряд элементов, которые следуют одно за другим или связаны друг за другом.
    • параллельно : Расположение электрических компонентов, при котором ток течет по двум или более путям.
    • Комбинированная схема : электрическая цепь, содержащая несколько резисторов, которые соединены как последовательным, так и параллельным соединением.

    Комбинированные схемы

    Более сложные соединения резисторов иногда представляют собой просто комбинации последовательного и параллельного. Это часто встречается, особенно если учитывать сопротивление проводов.В этом случае сопротивление провода включено последовательно с другими сопротивлениями, включенными параллельно.

    Комбинированная цепь может быть разбита на аналогичные части, которые являются последовательными или параллельными, как показано на схеме. На рисунке общее сопротивление может быть вычислено путем соединения трех резисторов друг с другом последовательно или параллельно. R 1 и R 2 соединены параллельно друг другу, поэтому мы знаем, что для этого подмножества сопротивление, обратное сопротивлению, будет равно:

    Сеть резисторов : В этой комбинированной схеме цепь может быть разбита на последовательный компонент и параллельный компонент.

    Комбинированные схемы : Два параллельных резистора, соединенные последовательно с одним резистором.

    [латекс] \ frac {1} {\ text {R} _1} + \ frac {1} {\ text {R} _2} [/ latex] или [латекс] \ frac {\ text {R} _1 \ text {R} _2} {\ text {R} _1 + \ text {R} _2} [/ latex]

    R 3 соединены последовательно с как R 1 , так и R 2 , поэтому сопротивление будет рассчитываться как:

    [латекс] \ text {R} = \ frac {\ text {R} _1 \ text {R} _2} {\ text {R} _1 + \ text {R} _2} + \ text {R} _3 [/ латекс ]

    Сложные комбинированные схемы

    Для более сложных комбинированных схем различные части могут быть идентифицированы как последовательные или параллельные, уменьшены до их эквивалентов, а затем уменьшены до тех пор, пока не останется единственное сопротивление, как показано на.На этом рисунке комбинация из семи резисторов была идентифицирована как включенные последовательно или параллельно. На исходном изображении две обведенные кружком секции показывают резисторы, включенные параллельно.

    Сокращение комбинированной схемы : Эта комбинация из семи резисторов имеет как последовательные, так и параллельные части. Каждое из них идентифицируется и приводится к эквивалентному сопротивлению, а затем уменьшается до тех пор, пока не будет достигнуто единичное эквивалентное сопротивление.

    Уменьшение этих параллельных резисторов до одного значения R позволяет нам визуализировать схему в более упрощенном виде.На верхнем правом изображении мы видим, что обведенная кружком часть содержит два последовательно соединенных резистора. Мы можем дополнительно уменьшить это до другого значения R, добавив их. Следующий шаг показывает, что два обведенных резистора включены параллельно. Уменьшение тех бликов, что последние два соединены последовательно и, таким образом, могут быть уменьшены до одного значения сопротивления для всей цепи.

    Практическое применение комбинированной схемы состоит в том, что сопротивление в проводах снижает ток и мощность, подаваемую на резистор.Комбинированная цепь может быть преобразована в последовательную цепь на основе понимания эквивалентного сопротивления параллельных ветвей комбинированной цепи. Последовательная цепь может использоваться для определения общего сопротивления цепи. По сути, сопротивление провода является последовательным с резистором. Таким образом, увеличивается общее сопротивление и уменьшается ток. Если сопротивление провода относительно велико, как в изношенном (или очень длинном) удлинителе, то эти потери могут быть значительными. Если потребляется большой ток, падение ИК-излучения в проводах также может быть значительным.

    Зарядка аккумулятора: последовательные и параллельные ЭДС

    При последовательном включении источников напряжения их ЭДС и внутренние сопротивления складываются; параллельно они остаются прежними.

    Цели обучения

    Сравнить сопротивления и электродвижущие силы для источников напряжения, подключенных с одинаковой и противоположной полярностью, последовательно и параллельно

    Основные выводы

    Ключевые моменты
    • ЭДС, соединенные последовательно с одинаковой полярностью, являются аддитивными и приводят к более высокой общей ЭДС.
    • Две ЭДС, соединенные последовательно с противоположной полярностью, имеют общую ЭДС, равную разнице между ними, и могут использоваться для зарядки источника с более низким напряжением.
    • Два источника напряжения с идентичными ЭДС, соединенные параллельно, имеют чистую ЭДС, эквивалентную одному источнику ЭДС, однако чистое внутреннее сопротивление меньше и, следовательно, дает более высокий ток.
    Ключевые термины
    • параллельно : Расположение электрических компонентов, при котором ток течет по двум или более путям.
    • электродвижущая сила : (ЭДС) — напряжение, генерируемое батареей или магнитной силой в соответствии с законом Фарадея. Она измеряется в вольтах, а не в ньютонах, и поэтому на самом деле не является силой.
    • серия : ряд элементов, которые следуют одно за другим или соединяются одно за другим.

    Когда используется более одного источника напряжения, они могут быть подключены последовательно или параллельно, аналогично резисторам в цепи.Когда источники напряжения включены последовательно в одном направлении, их внутренние сопротивления складываются, а их электродвижущая сила или ЭДС складываются алгебраически. Эти типы источников напряжения распространены в фонариках, игрушках и других приборах. Обычно ячейки включены последовательно, чтобы обеспечить большую суммарную ЭДС.

    Фонарик и лампочка : Последовательное соединение двух источников напряжения в одном направлении. Эта схема представляет собой фонарик с двумя последовательно включенными ячейками (источниками напряжения) и одной лампочкой (сопротивление нагрузки).

    Батарея — это соединение нескольких гальванических элементов. Однако недостатком такого последовательного соединения ячеек является то, что их внутреннее сопротивление увеличивается. Иногда это может быть проблематично. Например, если вы поместите в машину две батареи на 6 В вместо обычной батареи на 12 В, вы добавите как ЭДС, так и внутреннее сопротивление каждой батареи. Таким образом, у вас будет такая же ЭДС 12 В, хотя внутреннее сопротивление тогда будет удвоено, что вызовет у вас проблемы, когда вы захотите запустить двигатель.

    Но, если ячейки противостоят друг другу — например, когда одна вставляется в прибор задом наперед, — общая ЭДС меньше, так как это алгебраическая сумма отдельных ЭДС. Когда он перевернут, он создает ЭДС, которая противодействует другой, и приводит к разнице между двумя источниками напряжения.

    Зарядное устройство : представляет два источника напряжения, соединенных последовательно с противоположными ЭДС. Ток течет в направлении большей ЭДС и ограничивается суммой внутренних сопротивлений.(Обратите внимание, что каждая ЭДС представлена ​​на рисунке буквой E.) Зарядное устройство, подключенное к аккумулятору, является примером такого подключения. Зарядное устройство должно иметь большую ЭДС, чем батарея, чтобы через него протекал обратный ток.

    Когда два источника напряжения с одинаковыми ЭДС соединены параллельно и также подключены к сопротивлению нагрузки, общая ЭДС равна индивидуальной ЭДС. Но общее внутреннее сопротивление уменьшается, поскольку внутренние сопротивления параллельны. Таким образом, параллельное соединение может производить больший ток.

    Две идентичные ЭДС : Два источника напряжения с одинаковыми ЭДС (каждый помечен буквой E), подключенные параллельно, создают одинаковую ЭДС, но имеют меньшее общее внутреннее сопротивление, чем отдельные источники. Параллельные комбинации часто используются для подачи большего тока.

    ЭДС и напряжение на клеммах

    Выходное напряжение или напряжение на клеммах источника напряжения, такого как аккумулятор, зависит от его электродвижущей силы и внутреннего сопротивления.

    Цели обучения

    Выразите взаимосвязь между электродвижущей силой и напряжением на клеммах в форме уравнения

    Основные выводы

    Ключевые моменты
    • Электродвижущая сила (ЭДС) — это разность потенциалов источника при отсутствии тока.
    • Напряжение на клеммах — это выходное напряжение устройства, измеренное на его клеммах.
    • Напряжение на клеммах рассчитывается по формуле V = ЭДС — Ir.
    Ключевые термины
    • электродвижущая сила : (ЭДС) — напряжение, генерируемое батареей или магнитной силой в соответствии с законом Фарадея. Она измеряется в вольтах, а не в ньютонах, и поэтому на самом деле не является силой.
    • напряжение на клеммах : выходное напряжение устройства, измеренное на его клеммах.
    • разность потенциалов : разница в потенциальной энергии между двумя точками в электрическом поле; разница в заряде между двумя точками в электрической цепи; Напряжение.

    Когда вы забываете выключить автомобильные фары, они постепенно тускнеют по мере разрядки аккумулятора. Почему они просто не мигают, когда батарея разряжена? Их постепенное затемнение означает, что выходное напряжение батареи уменьшается по мере разряда батареи. Причина снижения выходного напряжения для разряженных или перегруженных батарей заключается в том, что все источники напряжения состоят из двух основных частей — источника электрической энергии и внутреннего сопротивления.

    Электродвижущая сила

    Все источники напряжения создают разность потенциалов и могут подавать ток, если подключены к сопротивлению. В небольшом масштабе разность потенциалов создает электрическое поле, которое воздействует на заряды, вызывая ток. Мы называем эту разность потенциалов электродвижущей силой (сокращенно ЭДС). ЭДС — это вообще не сила; это особый тип разности потенциалов источника при отсутствии тока. Единицы измерения ЭДС — вольты.

    Электродвижущая сила напрямую связана с источником разности потенциалов, например, с конкретной комбинацией химических веществ в батарее.Однако при протекании тока ЭДС отличается от выходного напряжения устройства. Напряжение на выводах батареи, например, меньше, чем ЭДС, когда батарея подает ток, и оно падает дальше, когда батарея разряжается или разряжается. Однако, если выходное напряжение устройства можно измерить без потребления тока, то выходное напряжение будет равно ЭДС (даже для сильно разряженной батареи).

    Напряжение на клеммах

    представляет схематическое изображение источника напряжения.Выходное напряжение устройства измеряется на его выводах и называется напряжением на выводах В . Напряжение на клеммах определяется уравнением:

    Схематическое изображение источника напряжения : Любой источник напряжения (в данном случае углеродно-цинковый сухой элемент) имеет ЭДС, связанную с источником разности потенциалов, и внутреннее сопротивление r, связанное с его конструкцией. (Обратите внимание, что сценарий E означает ЭДС.) Также показаны выходные клеммы, на которых измеряется напряжение на клеммах V.Поскольку V = ЭДС-Ir, напряжение на клеммах равно ЭДС, только если ток не течет.

    [латекс] \ text {V} = \ text {emf} — \ text {Ir} [/ latex],

    , где r — внутреннее сопротивление, а I — ток, протекающий во время измерения.

    I является положительным, если ток течет от положительного вывода. Чем больше ток, тем меньше напряжение на клеммах. Точно так же верно, что чем больше внутреннее сопротивление, тем меньше напряжение на клеммах.

    10.3: Последовательные и параллельные резисторы

    Цели обучения

    К концу раздела вы сможете:

    • Определите термин эквивалентное сопротивление
    • Рассчитайте эквивалентное сопротивление резисторов, включенных последовательно
    • Рассчитайте эквивалентное сопротивление резисторов, включенных параллельно

    В статье «Ток и сопротивление» мы описали термин «сопротивление» и объяснили основную конструкцию резистора. По сути, резистор ограничивает поток заряда в цепи и представляет собой омическое устройство, где \ (V = IR \).В большинстве схем имеется более одного резистора. Если несколько резисторов соединены вместе и подключены к батарее, ток, подаваемый батареей, зависит от эквивалентного сопротивления цепи.

    Эквивалентное сопротивление комбинации резисторов зависит как от их индивидуальных значений, так и от способа их подключения. Самыми простыми комбинациями резисторов являются последовательное и параллельное соединение (Рисунок \ (\ PageIndex {1} \)). В последовательной цепи выходной ток первого резистора течет на вход второго резистора; следовательно, ток в каждом резисторе одинаков.В параллельной цепи все выводы резистора на одной стороне резисторов соединены вместе, а все выводы на другой стороне соединены вместе. В случае параллельной конфигурации каждый резистор имеет одинаковое падение потенциала на нем, и токи через каждый резистор могут быть разными, в зависимости от резистора. Сумма отдельных токов равна току, протекающему по параллельным соединениям.

    Рисунок \ (\ PageIndex {1} \): (a) При последовательном соединении резисторов ток одинаков в каждом резисторе.(b) При параллельном соединении резисторов напряжение на каждом резисторе одинаковое.

    Резисторы серии

    Считается, что резисторы

    включены последовательно, когда ток течет через резисторы последовательно. Рассмотрим рисунок \ (\ PageIndex {2} \), на котором показаны три последовательно включенных резистора с приложенным напряжением, равным \ (V_ {ab} \). Поскольку заряды проходят только по одному пути, ток через каждый резистор одинаков. Эквивалентное сопротивление набора резисторов при последовательном соединении равно алгебраической сумме отдельных сопротивлений.

    Рисунок \ (\ PageIndex {2} \): (a) Три резистора, подключенные последовательно к источнику напряжения. (b) Исходная схема сокращается до эквивалентного сопротивления и источника напряжения.

    На рисунке \ (\ PageIndex {2} \) ток, идущий от источника напряжения, протекает через каждый резистор, поэтому ток через каждый резистор одинаков. Ток в цепи зависит от напряжения, подаваемого источником напряжения, и сопротивления резисторов. Для каждого резистора происходит падение потенциала, равное потере электрической потенциальной энергии при прохождении тока через каждый резистор.Согласно закону Ома, падение потенциала \ (V \) на резисторе при протекании через него тока рассчитывается по формуле \ (V = IR \), где \ (I \) — ток в амперах (\ (A \)), а \ (R \) — сопротивление в Ом \ ((\ Omega) \). N V_i = 0.\]

    Это уравнение часто называют законом петли Кирхгофа, который мы рассмотрим более подробно позже в этой главе. На рисунке \ (\ PageIndex {2} \) сумма падения потенциала каждого резистора и напряжения, подаваемого источником напряжения, должна равняться нулю:

    \ [\ begin {align *} V — V_1 — V_2 — V_3 & = 0, \\ [4pt] V & = V_1 + V_2 + V_3, \\ [4pt] & = IR_1 + IR_2 + IR_3, \ end { выровнять *} \]

    Решение для \ (I \)

    \ [\ begin {align *} I & = \ frac {V} {R_1 + R_2 + R_3} \\ [4pt] & = \ frac {V} {R_ {S}}.\ end {align *} \]

    Поскольку ток через каждый компонент одинаков, равенство можно упростить до эквивалентного сопротивления (\ (R_ {S} \)), которое представляет собой просто сумму сопротивлений отдельных резисторов.

    Эквивалентное сопротивление в последовательной цепи

    Любое количество резисторов может быть подключено последовательно. Если \ (N \) резисторы соединены последовательно, эквивалентное сопротивление равно

    . N R_i.\ label {серия эквивалентных сопротивлений} \]

    Одним из результатов подключения компонентов в последовательную цепь является то, что если что-то происходит с одним компонентом, это влияет на все остальные компоненты. Например, если несколько ламп подключены последовательно и одна лампа перегорела, все остальные лампы погаснут.

    Пример \ (\ PageIndex {1} \): эквивалентное сопротивление, ток и мощность в последовательной цепи

    Батарея с напряжением на клеммах 9 В подключена к цепи, состоящей из четырех последовательно соединенных резисторов \ (20 \, \ Omega \) и одного \ (10 ​​\, \ Omega \) (Рисунок \ (\ PageIndex {3 } \)).Предположим, что батарея имеет незначительное внутреннее сопротивление.

    1. Рассчитайте эквивалентное сопротивление цепи.
    2. Рассчитайте ток через каждый резистор.
    3. Рассчитайте падение потенциала на каждом резисторе.
    4. Определите общую мощность, рассеиваемую резисторами, и мощность, потребляемую батареей.
    Рисунок \ (\ PageIndex {3} \): простая последовательная схема с пятью резисторами.

    Стратегия

    В последовательной цепи эквивалентное сопротивление представляет собой алгебраическую сумму сопротивлений.2R \), а общая мощность, рассеиваемая резисторами, равна сумме мощности, рассеиваемой каждым резистором. Мощность, подаваемая батареей, можно найти с помощью \ (P = I \ epsilon \).

    Решение

    1. Эквивалентное сопротивление — это алгебраическая сумма сопротивлений (уравнение \ ref {серия эквивалентных сопротивлений}): \ [\ begin {align *} R_ {S} & = R_1 + R_2 + R_3 + R_4 + R_5 \\ [4pt ] & = 20 \, \ Омега + 20 \, \ Омега + 20 \, \ Омега + 20 \, \ Омега + 10 \, \ Омега = 90 \, \ Омега.2 (10 \, \ Omega) = 0,1 \, W, \ nonumber \] \ [P_ {рассеивается} = 0,2 \, W + 0,2 \, W + 0,2 \, W + 0,2 \, W + 0,1 \, W = 0,9 \, W, \ nonumber \] \ [P_ {источник} = I \ epsilon = (0,1 \, A) (9 \, V) = 0,9 \, W. \ nonumber \]

    Значение

    Есть несколько причин, по которым мы использовали бы несколько резисторов вместо одного резистора с сопротивлением, равным эквивалентному сопротивлению цепи. Возможно, резистора необходимого размера нет в наличии, или нам нужно отводить выделяемое тепло, или мы хотим минимизировать стоимость резисторов.Каждый резистор может стоить от нескольких центов до нескольких долларов, но при умножении на тысячи единиц экономия затрат может быть значительной.

    Упражнение \ (\ PageIndex {1} \)

    Некоторые гирлянды миниатюрных праздничных огней закорачиваются при перегорании лампочки. Устройство, вызывающее короткое замыкание, называется шунтом, который позволяет току течь по разомкнутой цепи. «Короткое замыкание» похоже на протягивание куска проволоки через компонент. Луковицы обычно сгруппированы в серию по девять луковиц.Если перегорает слишком много лампочек, в конечном итоге открываются шунты. Что вызывает это?

    Ответ

    Эквивалентное сопротивление девяти последовательно соединенных лампочек составляет 9 R . Ток равен \ (I = V / 9 \, R \). Если одна лампочка перегорит, эквивалентное сопротивление составит 8 R , и напряжение не изменится, но ток возрастет \ ((I = V / 8 \, R \). Чем больше лампочек перегорят, ток станет равным. В конце концов, ток становится слишком большим, что приводит к сгоранию шунта.№ Р_и. \]

  • Одинаковый ток протекает последовательно через каждый резистор.
  • Отдельные последовательно включенные резисторы не получают полное напряжение источника, а делят его. Общее падение потенциала на последовательной конфигурации резисторов равно сумме падений потенциала на каждом резисторе.
  • Параллельные резисторы

    На рисунке \ (\ PageIndex {4} \) показаны резисторы, включенные параллельно, подключенные к источнику напряжения. Резисторы включены параллельно, когда один конец всех резисторов соединен непрерывным проводом с незначительным сопротивлением, а другой конец всех резисторов также соединен друг с другом непрерывным проводом с незначительным сопротивлением.Падение потенциала на каждом резисторе одинаковое. Ток через каждый резистор можно найти с помощью закона Ома \ (I = V / R \), где напряжение на каждом резисторе постоянно. Например, автомобильные фары, радио и другие системы подключены параллельно, так что каждая подсистема использует полное напряжение источника и может работать полностью независимо. То же самое и с электропроводкой в ​​вашем доме или любом здании.

    Рисунок \ (\ PageIndex {4} \): Два резистора, подключенных параллельно источнику напряжения.(b) Исходная схема сокращается до эквивалентного сопротивления и источника напряжения.

    Ток, протекающий от источника напряжения на рисунке \ (\ PageIndex {4} \), зависит от напряжения, подаваемого источником напряжения, и эквивалентного сопротивления цепи. В этом случае ток течет от источника напряжения и попадает в переход или узел, где цепь разделяется, протекая через резисторы \ (R_1 \) и \ (R_2 \). По мере прохождения зарядов от батареи часть заряда проходит через резистор \ (R_1 \), а часть — через резистор \ (R_2 \).Сумма токов, текущих в переход, должна быть равна сумме токов, текущих из перехода:

    \ [\ sum I_ {in} = \ sum I_ {out}. {- 1}.{-1}. \ label {10.3} \]

    Это соотношение приводит к эквивалентному сопротивлению \ (R_ {P} \), которое меньше наименьшего из отдельных сопротивлений. Когда резисторы соединены параллельно, от источника течет больше тока, чем протекает для любого из них по отдельности, поэтому общее сопротивление ниже.

    Пример \ (\ PageIndex {2} \): Анализ параллельной цепи

    Три резистора \ (R_1 = 1,00 \, \ Omega \), \ (R_2 = 2,00 \, \ Omega \) и \ (R_3 = 2,00 \, \ Omega \) подключены параллельно.Параллельное соединение подключается к источнику напряжения \ (V = 3,00 \, V \).

    1. Какое эквивалентное сопротивление?
    2. Найдите ток, подаваемый источником в параллельную цепь.
    3. Рассчитайте токи в каждом резисторе и покажите, что в сумме они равны выходному току источника.
    4. Рассчитайте мощность, рассеиваемую каждым резистором.
    5. Найдите выходную мощность источника и покажите, что она равна общей мощности, рассеиваемой резисторами.

    Стратегия

    (a) Общее сопротивление для параллельной комбинации резисторов определяется с помощью уравнения \ ref {10.3}. (Обратите внимание, что в этих расчетах каждый промежуточный ответ отображается с дополнительной цифрой.)

    (b) Ток, подаваемый источником, можно найти из закона Ома, заменив \ (R_ {P} \) на полное сопротивление \ (I = \ frac {V} {R_ {P}} \).

    (c) Отдельные токи легко вычислить по закону Ома \ (\ left (I_i = \ frac {V_i} {R_i} \ right) \), поскольку каждый резистор получает полное напряжение.{-1} = 0,50 \, \ Omega. \ Nonumber \] Общее сопротивление с правильным количеством значащих цифр равно \ (R_ {eq} = 0,50 \, \ Omega \). Как и предполагалось, \ (R_ {P} \) меньше наименьшего индивидуального сопротивления.

  • Полный ток можно найти из закона Ома, заменив полное сопротивление \ (R_ {P} \). Это дает \ [I = \ frac {V} {R_ {P}} = \ frac {3.00 \, V} {0.50 \, \ Omega} = 6.00 \, A. \ nonumber \] Текущий I для каждого устройства намного больше, чем для тех же устройств, подключенных последовательно (см. предыдущий пример).Схема с параллельным соединением имеет меньшее общее сопротивление, чем резисторы, включенные последовательно.
  • Отдельные токи легко вычислить по закону Ома, поскольку каждый резистор получает полное напряжение. Таким образом, \ [I_1 = \ frac {V} {R_1} = \ frac {3.00 \, V} {1.00 \, \ Omega} = 3.00 \, A. \ nonumber \] Аналогично, \ [I_2 = \ frac {V } {R_2} = \ frac {3.00 \, V} {2.00 \, \ Omega} = 1.50 \, A \ nonumber \] и \ [I_3 = \ frac {V} {R_3} = \ frac {3.00 \, V } {2.00 \, \ Omega} = 1.50 \, A. \ nonumber \] Полный ток — это сумма отдельных токов: \ [I_1 + I_2 + I_3 = 6.2} {2.00 \, \ Omega} = 4.50 \, W. \ nonumber \]
  • Общую мощность также можно рассчитать несколькими способами. Выбор \ (P = IV \) и ввод общей текущей доходности \ [P = IV = (6.00 \, A) (3.00 \, V) = 18.00 \, W. \ nonumber \]
  • Значение

    Общая мощность, рассеиваемая резисторами, также 18,00 Вт:

    \ [P_1 + P_2 + P_3 = 9,00 \, W + 4,50 \, W + 4,50 \, W = 18,00 \, W. \ nonumber \]

    Обратите внимание, что общая мощность, рассеиваемая резисторами, равна мощности, подаваемой источником.

    Упражнение \ (\ PageIndex {2A} \)

    Рассмотрим одну и ту же разность потенциалов \ ((V = 3,00 \, V) \), приложенную к одним и тем же трем последовательно включенным резисторам. Будет ли эквивалентное сопротивление последовательной цепи больше, меньше или равно трем резисторам, включенным параллельно? Будет ли ток в последовательной цепи выше, ниже или равен току, обеспечиваемому тем же напряжением, приложенным к параллельной цепи? Как мощность, рассеиваемая последовательно подключенными резисторами, будет сравниваться с мощностью, рассеиваемой параллельно резисторами?

    Решение

    Эквивалент последовательной схемы будет \ (R_ {eq} = 1.00 \, \ Omega + 2.00 \, \ Omega + 2.00 \, \ Omega = 5.00 \, \ Omega \), что выше эквивалентного сопротивления параллельной цепи \ (R_ {eq} = 0.50 \, \ Omega \ ). Эквивалентный резистор любого количества резисторов всегда выше, чем эквивалентное сопротивление тех же резисторов, соединенных параллельно. Ток через последовательную цепь будет \ (I = \ frac {3.00 \, V} {5.00 \, \ Omega} = 0.60 \, A \), что меньше суммы токов, проходящих через каждый резистор в параллельная цепь, \ (I = 6.00 \, А \). Это неудивительно, поскольку эквивалентное сопротивление последовательной цепи выше. Ток при последовательном соединении любого количества резисторов всегда будет ниже, чем ток при параллельном соединении тех же резисторов, поскольку эквивалентное сопротивление последовательной цепи будет выше, чем параллельной цепи. Мощность, рассеиваемая последовательно подключенными резисторами, будет равна \ (P = 1,800 \, Вт \), что ниже мощности, рассеиваемой в параллельной цепи \ (P = 18.00 \, Вт \).

    Упражнение \ (\ PageIndex {2B} \)

    Как бы вы использовали реку и два водопада, чтобы смоделировать параллельную конфигурацию двух резисторов? Как разрушается эта аналогия?

    Решение

    Река, текущая горизонтально с постоянной скоростью, разделяется на две части и течет через два водопада. Молекулы воды аналогичны электронам в параллельных цепях. Количество молекул воды, которые текут в реке и падает, должно быть равно количеству молекул, которые текут над каждым водопадом, точно так же, как сумма тока через каждый резистор должна быть равна току, текущему в параллельном контуре.Молекулы воды в реке обладают энергией благодаря своему движению и высоте. Потенциальная энергия молекул воды в реке постоянна из-за их одинаковой высоты. Это аналогично постоянному изменению напряжения в параллельной цепи. Напряжение — это потенциальная энергия на каждом резисторе.

    При рассмотрении энергии аналогия быстро разрушается. В водопаде потенциальная энергия преобразуется в кинетическую энергию молекул воды. В случае прохождения электронов через резистор падение потенциала преобразуется в тепло и свет, а не в кинетическую энергию электронов.

    Суммируем основные характеристики резисторов параллельно:

    1. Эквивалентное сопротивление находится по формуле \ ref {10.3} и меньше любого отдельного сопротивления в комбинации.
    2. Падение потенциала на каждом параллельном резисторе одинаковое.
    3. Не каждый параллельный резистор получает полный ток; они делят это. Ток, поступающий в параллельную комбинацию резисторов, равен сумме токов, протекающих через каждый резистор, включенный параллельно.

    В этой главе мы представили эквивалентное сопротивление резисторов, соединенных последовательно, и резисторов, соединенных параллельно. Как вы помните, в разделе «Емкость» мы ввели эквивалентную емкость конденсаторов, соединенных последовательно и параллельно. Цепи часто содержат как конденсаторы, так и резисторы. Таблица \ (\ PageIndex {1} \) суммирует уравнения, используемые для эквивалентного сопротивления и эквивалентной емкости для последовательных и параллельных соединений.

    Таблица \ (\ PageIndex {1} \): сводка по эквивалентному сопротивлению и емкости в последовательной и параллельной комбинациях
    Комбинация серий Параллельная комбинация
    Эквивалентная емкость \ [\ frac {1} {C_ {S}} = \ frac {1} {C_1} + \ frac {1} {C_2} + \ frac {1} {C_3} +.N R_i \ nonumber \] \ [\ frac {1} {R_ {P}} = \ frac {1} {R_1} + \ frac {1} {R_2} + \ frac {1} {R_3} +. . . \ nonumber \]

    Сочетания последовательного и параллельного

    Более сложные соединения резисторов часто представляют собой просто комбинации последовательного и параллельного соединения. Такие комбинации обычны, особенно если учесть сопротивление проводов. В этом случае сопротивление провода включено последовательно с другими сопротивлениями, включенными параллельно.

    Комбинации последовательного и параллельного соединения можно свести к одному эквивалентному сопротивлению, используя технику, показанную на рисунке \ (\ PageIndex {5} \).Различные части могут быть идентифицированы как последовательные или параллельные соединения, уменьшенные до их эквивалентных сопротивлений, а затем уменьшенные до тех пор, пока не останется единственное эквивалентное сопротивление. Процесс занимает больше времени, чем труден. Здесь мы отмечаем эквивалентное сопротивление как \ (R_ {eq} \).

    Рисунок \ (\ PageIndex {5} \): (а) Исходная схема из четырех резисторов. (b) Шаг 1: резисторы \ (R_3 \) и \ (R_4 \) включены последовательно, и эквивалентное сопротивление равно \ (R_ {34} = 10 \, \ Omega \). (c) Шаг 2: сокращенная схема показывает, что резисторы \ (R_2 \) и \ (R_ {34} \) включены параллельно, с эквивалентным сопротивлением \ (R_ {234} = 5 \, \ Omega \).(d) Шаг 3: сокращенная схема показывает, что \ (R_1 \) и \ (R_ {234} \) включены последовательно с эквивалентным сопротивлением \ (R_ {1234} = 12 \, \ Omega \), которое является эквивалентное сопротивление \ (R_ {eq} \). (e) Уменьшенная схема с источником напряжения \ (V = 24 \, V \) с эквивалентным сопротивлением \ (R_ {eq} = 12 \, \ Omega \). Это приводит к току \ (I = 2 \, A \) от источника напряжения.

    Обратите внимание, что резисторы \ (R_3 \) и \ (R_4 \) включены последовательно. Их можно объединить в одно эквивалентное сопротивление.Один из методов отслеживания процесса — включить резисторы в качестве индексов. {- 1} = 5 \, \ Omega.\ nonumber \]

    Этот шаг процесса сокращает схему до двух резисторов, показанных на рисунке \ (\ PageIndex {5d} \). Здесь схема сводится к двум резисторам, которые в данном случае включены последовательно. Эти два резистора можно уменьшить до эквивалентного сопротивления, которое является эквивалентным сопротивлением цепи:

    \ [R_ {eq} = R_ {1234} = R_1 + R_ {234} = 7 \, \ Omega + 5 \ Omega = 12 \, \ Omega. \ nonumber \]

    Основная цель этого анализа схемы достигнута, и теперь схема сводится к одному резистору и одному источнику напряжения.

    Теперь мы можем проанализировать схему. Ток, обеспечиваемый источником напряжения, равен \ (I = \ frac {V} {R_ {eq}} = \ frac {24 \, V} {12 \, \ Omega} = 2 \, A \). Этот ток проходит через резистор \ (R_1 \) и обозначается как \ (I_1 \). Падение потенциала на \ (R_1 \) можно найти с помощью закона Ома:

    \ [V_1 = I_1R_1 = (2 \, A) (7 \, \ Omega) = 14 \, V. \ nonumber \]

    Глядя на рисунок \ (\ PageIndex {5c} \), это оставляет \ (24 \, V — 14 \, V = 10 \, V \) отбрасывать в параллельной комбинации \ (R_2 \) и \ ( R_ {34} \).Ток через \ (R_2 \) можно найти с помощью закона Ома:

    \ [I_2 = \ frac {V_2} {R_2} = \ frac {10 \, V} {10 \, \ Omega} = 1 \, A. \ nonumber \]

    Резисторы \ (R_3 \) и \ (R_4 \) включены последовательно, поэтому токи \ (I_3 \) и \ (I_4 \) равны

    .

    \ [I_3 = I_4 = I — I_2 = 2 \, A — 1 \, A = 1 \, A. \ nonumber \]

    Используя закон Ома, мы можем найти падение потенциала на двух последних резисторах. Потенциальные капли равны \ (V_3 = I_3R_3 = 6 \, V \) и \ (V_4 = I_4R_4 = 4 \, V \).2 (4 \, \ Omega) = 4 \, W, \\ [4pt] P_ {рассеивается} & = P_1 + P_2 + P_3 + P_4 = 48 \, W. \ end {align *} \]

    Общая энергия постоянна в любом процессе. Следовательно, мощность, подаваемая источником напряжения, составляет

    \ [\ begin {align *} P_s & = IV \\ [4pt] & = (2 \, A) (24 \, V) = 48 \, W \ end {align *} \]

    Анализ мощности, подаваемой в цепь, и мощности, рассеиваемой резисторами, является хорошей проверкой достоверности анализа; они должны быть равны.

    Пример \ (\ PageIndex {3} \): объединение последовательных и параллельных цепей

    На рисунке \ (\ PageIndex {6} \) показаны резисторы, подключенные последовательно и параллельно.Мы можем считать \ (R_1 \) сопротивлением проводов, ведущих к \ (R_2 \) и \ (R_3 \).

    1. Найдите эквивалентное сопротивление цепи.
    2. Какое падение потенциала \ (V_1 \) на резисторе \ (R_1 \)?
    3. Найдите ток \ (I_2 \) через резистор \ (R_2 \).
    4. Какая мощность рассеивается \ (R_2 \)?
    Рисунок \ (\ PageIndex {6} \): Эти три резистора подключены к источнику напряжения так, чтобы \ (R_2 \) и \ (R_3 \) были параллельны друг другу, и эта комбинация была последовательно с \ (R_1 \).

    Стратегия

    (a) Чтобы найти эквивалентное сопротивление, сначала найдите эквивалентное сопротивление параллельного соединения \ (R_2 \) и \ (R_3 \). Затем используйте этот результат, чтобы найти эквивалентное сопротивление последовательного соединения с \ (R_1 \).

    (b) Ток через \ (R_1 \) можно найти с помощью закона Ома и приложенного напряжения. Ток через \ (R_1 \) равен току от батареи. Падение потенциала \ (V_1 \) на резисторе \ (R_1 \) (которое представляет собой сопротивление в соединительных проводах) можно найти с помощью закона Ома.{-1} = 5.10 \, \ Omega. \ Nonumber \] Общее сопротивление этой комбинации является промежуточным между значениями чистой серии и чисто параллельной (\ (20.0 \, \ Omega \) и \ (0.804 \, \ Omega \) ), соответственно).

  • Ток через \ (R_1 \) равен току, обеспечиваемому батареей: \ [I_1 = I = \ frac {V} {R_ {eq}} = \ frac {12.0 \, V} {5.10 \, \ Omega} = 2.35 \, A. \ nonumber \] Напряжение на \ (R_1 \) равно \ [V_1 = I_1R_1 = (2.35 \, A) (1 \, \ Omega) = 2.35 \, V. \ nonumber \] Напряжение, приложенное к \ (R_2 \) и \ (R_3 \), меньше напряжения, подаваемого батареей, на величину \ (V_1 \).Когда сопротивление провода велико, это может существенно повлиять на работу устройств, представленных \ (R_2 \) и \ (R_3 \).
  • Чтобы найти ток через \ (R_2 \), мы должны сначала найти приложенное к нему напряжение. Напряжение на двух параллельных резисторах одинаковое: \ [V_2 = V_3 = V — V_1 = 12.0 \, V — 2.35 \, V = 9.65 \, V. \ nonumber \] Теперь мы можем найти ток \ (I_2 \) через сопротивление \ (R_2 \) по закону Ома: \ [I_2 = \ frac {V_2} {R_2} = \ frac {9.65 \, V} {6.00 \, \ Omega} = 1.2 (6.00 \, \ Omega) = 15.5 \, W. \ nonumber \]
  • Значение

    Анализ сложных схем часто можно упростить, сведя схему к источнику напряжения и эквивалентному сопротивлению. Даже если вся схема не может быть сведена к одному источнику напряжения и одному эквивалентному сопротивлению, части схемы могут быть уменьшены, что значительно упрощает анализ.

    Упражнение \ (\ PageIndex {3} \)

    Рассмотрите электрические цепи в вашем доме.Приведите по крайней мере два примера схем, которые должны использовать комбинацию последовательных и параллельных схем для эффективной работы.

    Решение

    Все цепи верхнего освещения параллельны и подключены к основному питанию, поэтому при перегорании одной лампочки все верхнее освещение не гаснет. У каждого верхнего света будет по крайней мере один переключатель, включенный последовательно с источником света, так что вы можете включать и выключать его.

    В холодильнике есть компрессор и лампа, которая загорается при открытии дверцы.Обычно для подключения холодильника к стене используется только один шнур. Цепь, содержащая компрессор, и цепь, содержащая цепь освещения, параллельны, но есть переключатель, включенный последовательно со светом. Термостат управляет переключателем, который включен последовательно с компрессором, чтобы контролировать температуру холодильника.

    Практическое применение

    Одним из следствий этого последнего примера является то, что сопротивление в проводах снижает ток и мощность, подаваемую на резистор.Если сопротивление провода относительно велико, как в изношенном (или очень длинном) удлинителе, то эти потери могут быть значительными. Если потребляется большой ток, падение IR в проводах также может быть значительным и может проявляться из-за тепла, выделяемого в шнуре.

    Например, когда вы роетесь в холодильнике и включается мотор, свет холодильника на мгновение гаснет. Точно так же вы можете увидеть тусклый свет в салоне, когда вы запускаете двигатель вашего автомобиля (хотя это может быть связано с сопротивлением внутри самой батареи).

    Что происходит в этих сильноточных ситуациях, показано на рисунке \ (\ PageIndex {7} \). Устройство, обозначенное символом \ (R_3 \), имеет очень низкое сопротивление, поэтому при его включении протекает большой ток. Этот увеличенный ток вызывает большее падение IR в проводах, обозначенных \ (R_1 \), снижая напряжение на лампочке (которое составляет \ (R_2 \)), которое затем заметно гаснет.

    Рисунок \ (\ PageIndex {7} \): Почему свет тускнеет, когда включен большой прибор? Ответ заключается в том, что большой ток, потребляемый двигателем прибора, вызывает значительное падение IR в проводах и снижает напряжение на свету.

    Стратегия решения проблем: последовательные и параллельные резисторы

    1. Нарисуйте четкую принципиальную схему, обозначив все резисторы и источники напряжения. Этот шаг включает список известных значений проблемы, поскольку они отмечены на вашей принципиальной схеме.
    2. Определите, что именно необходимо определить в проблеме (определите неизвестные). Письменный список полезен.
    3. Определите, включены ли резисторы последовательно, параллельно или в комбинации последовательно и параллельно.Изучите принципиальную схему, чтобы сделать эту оценку. Резисторы включены последовательно, если через них должен последовательно проходить один и тот же ток.
    4. Используйте соответствующий список основных функций для последовательных или параллельных соединений, чтобы найти неизвестные. Есть один список для серий, а другой — для параллелей.
    5. Проверьте, являются ли ответы разумными и последовательными.

    Пример \ (\ PageIndex {4} \): объединение последовательных и параллельных цепей

    Два резистора, соединенных последовательно \ ((R_1, \, R_2) \), соединены с двумя резисторами, включенными параллельно \ ((R_3, \, R_4) \).Последовательно-параллельная комбинация подключается к батарее. Каждый резистор имеет сопротивление 10,00 Ом. Провода, соединяющие резисторы и аккумулятор, имеют незначительное сопротивление. Через резистор \ (R_1 \) проходит ток 2,00 А. Какое напряжение подается от источника напряжения?

    Стратегия

    Используйте шаги предыдущей стратегии решения проблем, чтобы найти решение для этого примера.

    Решение

    Рисунок \ (\ PageIndex {8} \): Чтобы найти неизвестное напряжение, мы должны сначала найти эквивалентное сопротивление цепи.
    1. Нарисуйте четкую принципиальную схему (Рисунок \ (\ PageIndex {8} \)).
    2. Неизвестно напряжение аккумулятора. Чтобы определить напряжение, подаваемое батареей, необходимо найти эквивалентное сопротивление.
    3. В этой схеме мы уже знаем, что резисторы \ (R_1 \) и \ (R_2 \) включены последовательно, а резисторы \ (R_3 \) и \ (R_4 \) включены параллельно. Эквивалентное сопротивление параллельной конфигурации резисторов \ (R_3 \) и \ (R_4 \) последовательно с последовательной конфигурацией резисторов \ (R_1 \) и \ (R_2 \).{-1} = 5,00 \, \ Омега. \ nonumber \] Эта параллельная комбинация включена последовательно с двумя другими резисторами, поэтому эквивалентное сопротивление схемы равно \ (R_ {eq} = R_1 + R_2 + R_ {34} = (25.00 \, \ Omega \). поэтому напряжение, подаваемое батареей, равно \ (V = IR_ {eq} = 2.00 \, A (25.00 \, \ Omega) = 50.00 \, V \).
    4. Один из способов проверить соответствие ваших результатов — это рассчитать мощность, подаваемую батареей, и мощность, рассеиваемую резисторами. Мощность, обеспечиваемая аккумулятором, равна \ (P_ {batt} = IV = 100.2R_4 \\ [4pt] & = 40.00 \, W + 40.00 \, W + 10.00 \, W + 10.00 \, W = 100. \, W. \ end {align *} \]

      Поскольку мощность, рассеиваемая резисторами, равна мощности, выделяемой батареей, наше решение кажется последовательным.

      Значение

      Если проблема имеет комбинацию последовательного и параллельного соединения, как в этом примере, ее можно уменьшить поэтапно, используя предыдущую стратегию решения проблемы и рассматривая отдельные группы последовательных или параллельных соединений.При нахождении \ (R_ {eq} \) для параллельного соединения необходимо с осторожностью относиться к обратному. Кроме того, единицы и числовые результаты должны быть разумными. Эквивалентное последовательное сопротивление должно быть больше, а эквивалентное параллельное сопротивление, например, должно быть меньше. Мощность должна быть больше для одних и тех же устройств, подключенных параллельно, по сравнению с последовательными и т. Д.

      Авторы и авторство

      • Сэмюэл Дж. Линг (Государственный университет Трумэна), Джефф Санни (Университет Лойола Мэримаунт) и Билл Мобс со многими авторами.Эта работа лицензирована OpenStax University Physics в соответствии с лицензией Creative Commons Attribution License (4.0).

      Как рассчитать последовательные и параллельные резисторы — Kitronik Ltd

      Резисторы серии

      Когда резисторы подключаются друг за другом, это называется последовательным соединением. Это показано ниже. Чтобы рассчитать общее полное сопротивление ряда резисторов, подключенных таким образом, вы складываете отдельные сопротивления.Это делается по следующей формуле: Rtotal = R1 + R2 + R3 и так далее. Пример: чтобы рассчитать общее сопротивление для этих трех последовательно соединенных резисторов.
      Rtotal = R1 + R2 + R3 = 100 + 82 + 1 Ом = 183 Ом

      Задача 1:

      Рассчитайте общее сопротивление следующего последовательно включенного резистора.
      R Итого = _______________
      = _______________
      R Итого = _______________
      = _______________
      R Итого = _______________
      = _______________

      Параллельные резисторы

      Когда резисторы подключаются друг к другу (бок о бок), это называется параллельным подключением.Это показано ниже.

      Два параллельных резистора

      Для расчета общего полного сопротивления a двух резисторов, подключенных таким образом, вы можете использовать следующую формулу:
      Пример: чтобы рассчитать полное сопротивление для этих двух резисторов, включенных параллельно.

      Задача 2:

      Рассчитайте полное сопротивление следующего резистора, включенного параллельно.

      Три или более резистора, включенных параллельно

      Для расчета общего общего сопротивления ряда из трех или более резисторов, подключенных таким образом, вы можете использовать следующую формулу: Пример: Чтобы рассчитать общее сопротивление для этих трех резисторов, подключенных параллельно

      Задача 3:

      Рассчитайте полное сопротивление следующего резистора, включенного параллельно.

      ответов

      Задача 1

      1 = 1492 Ом 2 = 2242 Ом 3 = 4847 Ом

      Задача 2

      1 = 5 Ом 2 = 9,57 Ом 3 = 248,12 Ом

      Задача 3

      1 = 5,95 Ом 2 = 23,76 Ом Загрузите pdf-версию этой страницы здесь. Узнать больше об авторе подробнее »

      © Kitronik Ltd — Вы можете распечатать эту страницу и ссылку на нее, но не должны копировать страницу или ее часть без предварительного письменного согласия Kitronik.

      Учебное пособие по физике: Параллельные схемы

      Как упоминалось в предыдущем разделе Урока 4, два или более электрических устройства в цепи могут быть соединены последовательным или параллельным соединением.Когда все устройства соединены с использованием параллельных соединений, схема называется параллельной цепью . В параллельной схеме каждое устройство помещается в свою отдельную ветвь . Наличие ответвлений означает, что существует несколько путей, по которым заряд может проходить через внешнюю цепь. Каждый заряд, проходящий через контур внешней цепи, будет проходить через единственный резистор, присутствующий в одной ветви. По прибытии в место разветвления или узел плата делает выбор относительно того, через какую ветвь пройти на обратном пути к терминалу с низким потенциалом.

      Краткое сравнение и контраст между последовательными и параллельными цепями было сделано в предыдущем разделе Урока 4. В этом разделе было подчеркнуто, что добавление большего количества резисторов в параллельную цепь приводит к довольно неожиданному результату — уменьшению общего сопротивления. . Поскольку существует несколько путей, по которым может протекать заряд, добавление еще одного резистора в отдельную ветвь обеспечивает еще один путь, по которому заряд может проходить через основную область сопротивления в цепи.Это уменьшенное сопротивление в результате увеличения количества ветвей будет иметь эффект увеличения скорости, с которой течет заряд (также известной как ток). Чтобы сделать этот довольно неожиданный результат более разумным, была введена аналогия с платными дорогами. Плата за проезд — это основное место сопротивления автомобильному потоку на платной дороге. Добавление дополнительных пунктов сбора платы за проезд в пределах их собственного отделения на платной дороге обеспечит больше путей для автомобилей, проезжающих через станцию ​​сбора платы за проезд. Эти дополнительные пункты пропуска снизят общее сопротивление потоку автомобилей и увеличат скорость их движения.

      Текущий

      Скорость, с которой заряд проходит через цепь, называется током. Заряд НЕ накапливается и не начинает накапливаться в любом заданном месте, так что ток в одном месте больше, чем в других местах. Заряд НЕ расходуется резисторами таким образом, что в одном месте ток меньше, чем в другом. В параллельной схеме заряд делит на отдельные ветви, так что в одной ветви может быть больше тока, чем в другой.Тем не менее, если брать в целом, общая сумма тока во всех ветвях при сложении равна величине тока в местах за пределами ветвей. Правило, что ток везде одинаковый все еще работает, только с закруткой. Сила тока вне ветвей равна сумме токов в отдельных ветвях. Это все еще та же величина тока, только разделенная на несколько путей.

      В форме уравнения этот принцип можно записать как

      I итого = I 1 + I 2 + I 3 +…

      , где I total — общая сумма тока вне ветвей (и в батарее), а I 1 , I 2 и I 3 представляют ток в отдельных ветвях цепи.

      В этом блоке широко использовалась аналогия между расходом заряда и расходом воды. Еще раз вернемся к аналогии, чтобы проиллюстрировать, как сумма текущих значений в ветвях равна сумме вне ветвей.Поток заряда в проводах аналогичен потоку воды в трубах. Рассмотрим приведенные ниже схемы, на которых поток воды в трубах делится на отдельные ответвления. В каждом узле (место разветвления) вода проходит двумя или более отдельными путями. Скорость, с которой вода поступает в узел (измеряется в галлонах в минуту), будет равна сумме расходов в отдельных ветвях за пределами узла. Точно так же, когда две или более ветви подаются в узел, скорость, с которой вода вытекает из узла, будет равна сумме расходов в отдельных ветвях, которые подаются в узел.

      Тот же принцип разделения потока применяется к электрическим цепям. Скорость, с которой заряд поступает в узел, равна сумме расходов в отдельных ветвях за пределами узла. Это проиллюстрировано в приведенных ниже примерах. В примерах вводится новый символ схемы — буква A, заключенная в круг. Это символ амперметра — устройства, используемого для измерения силы тока в определенной точке. Амперметр способен измерять ток, оказывая при этом незначительное сопротивление потоку заряда.

      На диаграмме A показаны два резистора, подключенных параллельно узлам в точках A и B. Заряд течет в точку A со скоростью 6 ампер и делится на два пути — один через резистор 1, а другой через резистор 2. Ток в ветви с резистором 1 — 2 ампера, а ток в ветви с резистором 2 — 4 ампера. После того, как эти две ветви снова встретятся в точке B, чтобы сформировать единую линию, ток снова станет равным 6 ампер. Таким образом, мы видим, что принцип, согласно которому ток вне ветвей равен сумме тока в отдельных ветвях, справедлив.

      I итого = I 1 + I 2

      6 ампер = 2 ампера + 4 ампера

      Схема B выше может быть немного сложнее, если три резистора расположены параллельно. На схеме обозначены четыре узла, обозначенные буквами A, B, C и D. Заряд течет в точку A со скоростью 12 ампер и делится на два пути: один проходит через резистор 1, а другой направляется к точке B (и резисторам 2). и 3). 12 ампер тока делятся на 2-амперную (через резистор 1) и 10-амперную (в направлении точки B).В точке B происходит дальнейшее разделение потока на два пути — один через резистор 2, а другой через резистор 3. Ток в 10 ампер, приближающийся к точке B, делится на 6-амперный путь (через резистор 2) и 4-канальный. -амперный тракт (через резистор 3). Таким образом, видно, что значения тока в трех ветвях составляют 2 ампера, 6 ампер и 4 ампера, и что сумма значений тока в отдельных ветвях равна току вне ветвей.

      I итого = I 1 + I 2 + I 3

      12 ампер = 2 ампер + 6 ампер + 4 ампер

      Анализ потока в точках C и D также может быть проведен, и будет замечено, что сумма расходов потока в этих точках равна скорости потока, находящейся непосредственно за этими точками.

      Эквивалентное сопротивление

      Фактическая величина тока всегда обратно пропорциональна величине общего сопротивления. Существует четкая взаимосвязь между сопротивлением отдельных резисторов и общим сопротивлением набора резисторов. Чтобы исследовать эту взаимосвязь, давайте начнем с простейшего случая, когда два резистора помещены в параллельные ветви, каждый из которых имеет одинаковое значение сопротивления 4 Ом.Поскольку схема предлагает два равных путей для потока заряда, только половина заряда выберет для прохождения через данную ветвь. В то время как каждая отдельная ветвь предлагает сопротивление 4 Ом любому заряду, который проходит через нее, только половина всего заряда, протекающего по цепи, будет встречать сопротивление 4 Ом этой отдельной ветви. Таким образом, что касается батареи, которая накачивает заряд, наличие двух параллельно подключенных резисторов 4 Ом было бы эквивалентно наличию одного резистора 2 Ом в цепи.Таким же образом, наличие двух параллельно подключенных резисторов сопротивлением 6 Ом было бы эквивалентно наличию в цепи одного резистора сопротивлением 3 Ом. А наличие двух параллельных резисторов 12 Ом было бы эквивалентно наличию в цепи одного резистора 6 Ом.

      Теперь давайте рассмотрим другой простой случай, когда три резистора включены параллельно, каждый из которых имеет одинаковое сопротивление 6 Ом. При трех равных путях прохождения заряда через внешнюю цепь только одна треть заряда будет проходить через данную ветвь.Каждая отдельная ветвь обеспечивает сопротивление 6 Ом проходящему через нее заряду. Однако тот факт, что только одна треть заряда проходит через определенную ветвь, означает, что общее сопротивление цепи составляет 2 Ом. Что касается батареи, которая нагнетает заряд, наличие трех параллельных резисторов 6 Ом было бы эквивалентно наличию одного резистора 2 Ом в цепи. Таким же образом, наличие трех параллельно подключенных резисторов сопротивлением 9 Ом было бы эквивалентно наличию в цепи одного резистора сопротивлением 3 Ом.А наличие трех параллельных резисторов 12 Ом было бы эквивалентно наличию одного резистора 4 Ом в цепи.

      Это концепция эквивалентного сопротивления. Эквивалентное сопротивление схемы — это величина сопротивления, которая потребуется одному резистору, чтобы сравняться с общим эффектом от набора резисторов, присутствующих в схеме. Для параллельных цепей математическая формула для вычисления эквивалентного сопротивления (R eq ) составляет

      . 1 / R экв. = 1 / R 1 + 1 / R 2 + 1 / R 3 +…

      , где R 1 , R 2 и R 3 — значения сопротивления отдельных резисторов, подключенных параллельно. Приведенные выше примеры можно рассматривать как простые случаи, в которых все пути обладают одинаковым сопротивлением отдельному заряду, который проходит через них. Приведенные выше простые случаи были выполнены без использования уравнения. Тем не менее, уравнение подходит как для простых случаев, когда резисторы ответвления имеют одинаковые значения сопротивления, так и для более сложных случаев, когда резисторы ответвления имеют разные значения сопротивления.Например, рассмотрим применение уравнения к одному простому и одному сложному случаю ниже.

      Случай 1 : Три резистора 12 Ом включены параллельно

      1 / R экв = 1 / R 1 + 1 / R 2 + 1 / R 3

      1 / R экв = 1 / (12 Ом) + 1 / (12 Ом) + 1 / (12 Ом)

      Использование калькулятора …

      1 / R экв. = 0,25 Ом -1

      R экв = 1 / (0,25 Ом -1 )

      R экв = 4,0 Ом

      Случай 2 : резисторы 5,0 Ом, 7,0 Ом и 12 Ом включены параллельно

      1 / R экв = 1 / R 1 + 1 / R 2 + 1 / R 3

      1 / R экв = 1 / (5.0 Ом) + 1 / (7,0 Ом) + 1 / (12 Ом)

      Использование калькулятора …

      1 / R экв = 0,42619 Ом-1

      R экв = 1 / (0,42619 Ом -1 )

      R экв = 2,3 Ом


      Ваша очередь попробовать

      Нужно больше практики? Используйте Два параллельных резистора виджет ниже, чтобы попробовать некоторые дополнительные проблемы.Введите любые два желаемых значения сопротивления. Воспользуйтесь калькулятором, чтобы определить значения рэндов и рэндов. Затем нажмите кнопку Отправить , чтобы проверить свои ответы. Попробуйте столько раз, сколько хотите, с разными значениями сопротивления.

      Падения напряжения в параллельных ответвлениях

      В разделе «Схемы» учебного пособия «Физический класс» подчеркивалось, что любое повышение напряжения, полученное за счет заряда в батарее, теряется из-за заряда, когда он проходит через резисторы внешней цепи.Общее падение напряжения во внешней цепи равно увеличению напряжения при прохождении заряда по внутренней цепи. В параллельной схеме заряд не проходит через каждый резистор; скорее, он проходит через единственный резистор. Таким образом, полное падение напряжения на этом резисторе должно соответствовать напряжению батареи. Не имеет значения, проходит ли заряд через резистор 1, резистор 2 или резистор 3, падение напряжения на резисторе, которое выбирает для прохождения, должно равняться напряжению батареи.В форме уравнения этот принцип можно было бы выразить как

      В аккумулятор = В 1 = В 2 = В 3 = …

      Если три резистора размещены в параллельных ветвях и питаются от 12-вольтовой батареи, то падение напряжения на каждом из трех резисторов составляет 12 вольт. Заряд, протекающий по цепи, встретит только один из этих трех резисторов и, таким образом, столкнется с одним падением напряжения на 12 вольт.

      Диаграммы электрических потенциалов были представлены в Уроке 1 этого устройства и впоследствии использовались для иллюстрации последовательных падений напряжения, происходящих в последовательных цепях.Диаграмма электрических потенциалов — это концептуальный инструмент для представления разности электрических потенциалов между несколькими точками электрической цепи. Рассмотрим приведенную ниже принципиальную схему и соответствующую диаграмму электрических потенциалов.

      Как показано на диаграмме электрических потенциалов, все позиции A, B, C, E и G имеют высокий электрический потенциал. Один заряд выбирает только один из трех возможных путей; таким образом, в позиции B один заряд будет двигаться к точкам C, E или G, а затем пройдет через резистор, находящийся в этой ветви.Заряд не теряет свой высокий потенциал до тех пор, пока он не пройдет через резистор, либо от C к D, от E к F или от G к H. После того, как он пройдет через резистор, заряд вернется почти до 0 вольт и вернется к отрицательному значению. клемму аккумуляторной батареи для повышения ее напряжения. В отличие от последовательных цепей, заряд в параллельной цепи встречает единственное падение напряжения на своем пути через внешнюю цепь.

      Ток через заданную ветвь можно предсказать, используя уравнение закона Ома, падение напряжения на резисторе и сопротивление резистора.Поскольку падение напряжения на каждом резисторе одинаковое, фактором, определяющим, что резистор имеет наибольший ток, является сопротивление. Резистор с наибольшим сопротивлением испытывает наименьший ток, а резистор с наименьшим сопротивлением — наибольший ток. В этом смысле можно сказать, что заряд (как и люди) выбирает путь наименьшего сопротивления. В форме уравнения это может быть указано как

      I 1 = Δ V 1 / R 1 I 2 = Δ V 2 / R 2 I 3 = Δ V 3 / R 3

      Этот принцип иллюстрируется схемой, показанной ниже.Произведение I • R одинаково для каждого резистора (и равно напряжению батареи). Тем не менее, ток у каждого резистора разный. Ток наибольший там, где сопротивление наименьшее, и ток наименьший, где сопротивление наибольшее.

      Математический анализ параллельных цепей

      Приведенные выше принципы и формулы могут использоваться для анализа параллельной цепи и определения значений тока и разности электрических потенциалов на каждом из резисторов в параллельной цепи.Их использование будет продемонстрировано математическим анализом схемы, показанной ниже. Цель состоит в том, чтобы использовать формулы для определения эквивалентного сопротивления цепи (R eq ), тока через батарею (I до ), а также падений напряжения и тока для каждого из трех резисторов.

      Анализ начинается с использования значений сопротивления отдельных резисторов для определения эквивалентного сопротивления цепи.

      1 / R экв = 1 / R 1 + 1 / R 2 + 1 / R 3 = (1/17 Ω) + (1/12 Ω) + (1/11 Ω)

      1 / R экв = 0.23306 Ом -1

      R экв = 1 / (0,23306 Ом -1 )

      R экв = 4,29 Ом

      (округлено от 4,29063 Ом)

      Теперь, когда известно эквивалентное сопротивление, ток в батарее можно определить с помощью уравнения закона Ома. При использовании уравнения закона Ома (ΔV = I • R) для определения тока в батарее важно использовать напряжение батареи для ΔV и эквивалентное сопротивление для R.Расчет показан здесь:

      I до = ΔV аккумулятор / R eq = (60 В) / (4,29063 Ом)

      I до = 14,0 А

      (округлено от 13,98396 А)

      Напряжение батареи 60 В представляет собой усиление электрического потенциала за счет заряда, проходящего через батарею. Заряд теряет такое же количество электрического потенциала при любом прохождении через внешнюю цепь.То есть падение напряжения на каждом из трех резисторов такое же, как и напряжение, полученное в батарее:

      ΔV аккумулятор = ΔV 1 = ΔV 2 = ΔV 3 = 60 В

      Осталось определить три значения — ток каждого отдельного резистора. Закон Ома снова используется для определения значений тока для каждого резистора — это просто падение напряжения на каждом резисторе (60 В), деленное на сопротивление каждого резистора (указанное в формулировке задачи).Расчеты показаны ниже.

      I 1 = ΔV 1 / R 1

      I 1 = (60 В) / (17 Ом)

      I 1 = 3,53 А

      I 2 = ΔV 2 / R 2

      I 2 = (60 В) / (12 Ом)

      I 2 = 5,00 А

      I 3 = ΔV 3 / R 3

      I 3 = (60 В) / (11 Ом)

      Я 3 = 5.45 ампер

      Для проверки точности выполненных математических расчетов целесообразно проверить, удовлетворяют ли вычисленные значения принципу, согласно которому сумма значений тока для каждого отдельного резистора равна общему току в цепи (или в батарее). . Другими словами, I до = I 1 + I 2 + I 3 ?

      Я = Я 1 + Я 2 + Я 3 ?

      Из 14.0 ампер = 3,53 ампер + 5,00 ампер + 5,45 ампер?

      14,0 А = 13,98 А?

      Да !!

      (Разница в 0,02 ампера — это просто результат предыдущего округления значения I до от 13,98.)

      Математический анализ этой параллельной цепи включал смесь концепций и уравнений. Как это часто бывает в физике, отделение понятий от уравнений при принятии решения физической проблемы является опасным актом.Здесь необходимо учитывать концепции, согласно которым падение напряжения на каждом из трех резисторов равно напряжению батареи и что сумма тока в каждом резисторе равна общему току. Эти представления необходимы для завершения математического анализа. В следующей части Урока 4 будут исследованы комбинированные или составные схемы, в которых одни устройства включены параллельно, а другие — последовательно.

      Создавайте, решайте и проверяйте свои собственные проблемы с помощью виджета Equivalent Resistance ниже.Создайте себе проблему с любым количеством резисторов и любыми номиналами. Решать проблему; затем нажмите кнопку «Отправить», чтобы проверить свой ответ.

      Мы хотели бы предложить … Зачем просто читать об этом и когда можно с этим взаимодействовать? Взаимодействовать — это именно то, что вы делаете, когда используете одно из интерактивных материалов The Physics Classroom. Мы хотели бы предложить вам совместить чтение этой страницы с использованием нашего интерактивного средства построения цепей постоянного тока.Вы можете найти его в разделе Physics Interactives на нашем сайте. Построитель цепей постоянного тока предоставляет учащемуся набор для построения виртуальных цепей. Вы можете легко перетащить источники напряжения, резисторы и провода на рабочее место, расположить и подключить их так, как вам нужно. Вольтметры и амперметры позволяют измерять ток и падение напряжения. Нажатие на резистор или источник напряжения позволяет изменять сопротивление или входное напряжение. Это просто. Это весело. И это безопасно (если вы не используете его в ванне).


      Проверьте свое понимание

      1. По мере того, как в цепь добавляется все больше и больше резисторов, эквивалентное сопротивление цепи ____________ (увеличивается, уменьшается) и общий ток цепи ____________ (увеличивается, уменьшается).

      2.Три одинаковых лампочки подключены к D-ячейке, как показано ниже. P, Q, X, Y и Z обозначают местоположения вдоль цепи. Какое из следующих утверждений верно?

      а. Ток в точке Y больше, чем ток в точке Q.

      г. Ток на Y больше, чем на P.

      .

      г. Ток в точке Y больше, чем ток в точке Z.

      г. Ток в точке P больше, чем ток в точке Q.

      .

      e.Ток на Q больше, чем на P.

      ф. Сила тока одинакова во всех местах.

      3. Три одинаковые лампочки подключены к D-ячейке, как показано ниже. P, Q, X, Y и Z обозначают местоположения вдоль цепи. В каком (их) месте (ах) ток будет …

      а. … так же, как у X?

      г…. такой же, как у Q?

      г. … так же, как у Y?

      г. … меньше, чем у Q?

      e. … меньше, чем у P?

      ф. … вдвое больше, чем у Z?

      г. … в три раза больше, чем в Y?

      4. Какие изменения можно внести в схему ниже, чтобы уменьшить ток в ячейке? Перечислите все подходящие варианты.

      а. Увеличьте сопротивление лампы X.

      г. Уменьшите сопротивление лампы X.

      г. Увеличьте сопротивление лампы Z.

      г. Уменьшите сопротивление лампы Z.

      .

      e. Увеличьте напряжение ячейки (как-нибудь).

      ф. Уменьшите напряжение ячейки (как-нибудь).

      г. Снять лампу Y.

      .

      5.Аккумулятор на 12 В, резистор на 12 Ом и резистор на 4 Ом подключаются, как показано на рисунке. Ток в резисторе 12 Ом равен ____ току в резисторе 4 Ом.

      а. 1/3

      г. 1/2

      г. 2/3

      г. то же, что

      e.1,5 раза

      ф. дважды

      г. трижды

      ч. четыре раза


      6. Аккумулятор на 12 В, резистор на 12 Ом и резистор на 4 Ом подключены, как показано.Падение напряжения на резисторе 12 Ом равно ____ падению напряжения на резисторе 4 Ом.

      а. 1/3

      г. 1/2

      г. 2/3

      г. то же, что

      e. 1,5 раза

      ф.дважды

      г. трижды

      ч. четыре раза

      7. Аккумулятор на 12 В и резистор на 12 Ом подключаются, как показано на схеме. Резистор на 6 Ом добавлен к резистору на 12 Ом, чтобы создать цепь Y, как показано.Падение напряжения на резисторе 6 Ом в цепи Y равно ____ падению напряжения на резисторе в цепи X.

      а. больше, чем

      г. меньше

      г. то же, что

      8. Используйте свое понимание эквивалентного сопротивления, чтобы заполнить следующие утверждения:

      а. Два резистора сопротивлением 6 Ом, помещенные параллельно, обеспечат сопротивление, эквивалентное сопротивлению одного резистора _____ Ом.

      г. Три резистора 6 Ом, помещенные параллельно, обеспечат сопротивление, эквивалентное одному резистору _____ Ом.

      г. Три резистора сопротивлением 8 Ом, помещенные параллельно, обеспечат сопротивление, эквивалентное сопротивлению одного резистора _____ Ом.

      г. Три резистора с сопротивлением 2 Ом, 4 Ом и 6 Ом размещены параллельно. Они обеспечили бы сопротивление, эквивалентное одному резистору _____ Ом.

      e. Три резистора с сопротивлением 5 Ом, 6 Ом и 7 Ом размещены параллельно.Они обеспечили бы сопротивление, эквивалентное одному резистору _____ Ом.

      ф. Три резистора с сопротивлением 12 Ом, 6 Ом и 21 Ом размещены параллельно. Они обеспечили бы сопротивление, эквивалентное одному резистору _____ Ом.

      9. На основании ваших ответов на вышеуказанный вопрос заполните следующую формулировку:

      Общее или эквивалентное сопротивление трех параллельно включенных резисторов будет _____.

      а. больше, чем сопротивление самого большого значения R.

      г. меньше, чем сопротивление наименьшего значения R из трех.

      г. где-то между наименьшим значением R и наибольшим значением R.

      г. … бред какой то! Такого обобщения сделать нельзя. Результаты меняются.

      10. Три резистора включены параллельно.При размещении в цепи с источником питания 12 В. Определите эквивалентное сопротивление, полный ток цепи, падение напряжения и ток в каждом резисторе.


      Учебное пособие по физике: Комбинированные схемы

      Ранее в Уроке 4 упоминалось, что существует два разных способа соединения двух или более электрических устройств в цепь.Они могут быть соединены посредством последовательного или параллельного соединения. Когда все устройства в цепи соединены последовательными соединениями, тогда схема называется последовательной схемой. Когда все устройства в цепи соединены параллельными соединениями, тогда схема называется параллельной цепью. Третий тип схемы предполагает двойное использование последовательного и параллельного соединений в схеме; такие схемы называются составными схемами или комбинированными схемами.Схема, изображенная справа, является примером использования как последовательного, так и параллельного соединения в одной цепи. В этом случае лампочки A и B подключаются параллельно, а лампочки C и D подключаются последовательно. Это пример комбинированной схемы .

      При анализе комбинированных цепей критически важно иметь твердое понимание концепций, относящихся как к последовательным цепям, так и к параллельным цепям.Поскольку оба типа соединений используются в комбинированных схемах, концепции, связанные с обоими типами схем, применяются к соответствующим частям схемы. Основные понятия, связанные с последовательными и параллельными цепями, представлены в таблице ниже.

      Цепи серии
      • Ток одинаков на всех резисторах; этот ток равен току в батарее.
      • Сумма падений напряжения на отдельных резисторах равна номинальному напряжению батареи.
      • Общее сопротивление набора резисторов равно сумме отдельных значений сопротивлений,
      R до = 1 R 2 R 3 + …
      Параллельные схемы
      • Падение напряжения одинаково на каждой параллельной ветви.
      • Сумма тока в каждой отдельной ветви равна току вне ветвей.
      • Эквивалентное или полное сопротивление набора резисторов определяется уравнением 1 / R экв. = 1 / R 1 + 1 / R 2 + 1 / R 3

      Каждое из вышеперечисленных понятий имеет математическое выражение. Комбинирование математических выражений вышеуказанных понятий с уравнением закона Ома (ΔV = I • R) позволяет провести полный анализ комбинированной схемы.

      Анализ комбинированных схем

      Основная стратегия анализа комбинированных схем включает использование значения эквивалентного сопротивления для параллельных ветвей для преобразования комбинированной схемы в последовательную. После преобразования в последовательную схему анализ можно проводить обычным образом. Ранее в Уроке 4 описывался метод определения эквивалентного параллельного сопротивления, затем общее или эквивалентное сопротивление этих ветвей равно сопротивлению одной ветви, деленному на количество ветвей.

      Этот метод соответствует формуле

      1 / R экв. = 1 / R 1 + 1 / R 2 + 1 / R 3 + …

      , где R 1 , R 2 и R 3 — значения сопротивления отдельных резисторов, подключенных параллельно. Если два или более резистора, находящиеся в параллельных ветвях, не имеют одинакового сопротивления, необходимо использовать приведенную выше формулу.Пример этого метода был представлен в предыдущем разделе Урока 4.

      Применяя свое понимание эквивалентного сопротивления параллельных ветвей к комбинированной схеме, комбинированную схему можно преобразовать в последовательную. Затем понимание эквивалентного сопротивления последовательной цепи можно использовать для определения общего сопротивления цепи. Рассмотрим следующие диаграммы ниже. Схема A представляет собой комбинированную схему с резисторами R 2 и R 3 , размещенными в параллельных ветвях.Два параллельных резистора 4 Ом эквивалентны сопротивлению 2 Ом. Таким образом, две ветви можно заменить одним резистором с сопротивлением 2 Ом. Это показано на диаграмме B. Теперь, когда все резисторы включены последовательно, можно использовать формулу для общего сопротивления последовательных резисторов для определения общего сопротивления этой цепи: Формула для последовательного сопротивления составляет

      . R до = 1 R 2 R 3 + …

      Итак, на схеме B полное сопротивление цепи составляет 10 Ом.

      После определения общего сопротивления цепи анализ продолжается с использованием закона Ома и значений напряжения и сопротивления для определения значений тока в различных местах. Весь метод проиллюстрирован ниже на двух примерах.

      Пример 1:

      Первый пример — самый простой — резисторы, включенные параллельно, имеют одинаковое сопротивление. Цель анализа — определить ток и падение напряжения на каждом резисторе.

      Как обсуждалось выше, первым шагом является упрощение схемы путем замены двух параллельных резисторов одним резистором с эквивалентным сопротивлением. Два последовательно подключенных резистора 8 Ом эквивалентны одному резистору 4 Ом. Таким образом, два резистора ответвления (R 2 и R 3 ) можно заменить одним резистором с сопротивлением 4 Ом. Этот резистор 4 Ом включен последовательно с R 1 и R 4 . Таким образом, общее сопротивление составляет

      . R до = R 1 + 4 Ом + R 4 = 5 Ом + 4 Ом + 6 Ом

      R до = 15 Ом

      Теперь уравнение закона Ома (ΔV = I • R) можно использовать для определения полного тока в цепи.При этом необходимо использовать общее сопротивление и общее напряжение (или напряжение батареи).

      I tot = ΔV tot / R tot = (60 В) / (15 Ом)

      I до = 4 А

      Расчет тока 4 А представляет собой ток в месте расположения батареи. При этом резисторы R 1 и R 4 включены последовательно, а ток в последовательно соединенных резисторах везде одинаков.Таким образом,

      I до = I 1 = I 4 = 4 А

      Для параллельных ветвей сумма тока в каждой отдельной ветви равна току вне ветвей. Таким образом, I 2 + I 3 должно равняться 4 ампер. Существует бесконечное количество возможных значений I 2 и I 3 , которые удовлетворяют этому уравнению. Поскольку значения сопротивления равны, значения тока в этих двух резисторах также равны.Следовательно, ток в резисторах 2 и 3 равен 2 А.

      I 2 = I 3 = 2 А

      Теперь, когда известен ток в каждом отдельном месте резистора, можно использовать уравнение закона Ома (ΔV = I • R) для определения падения напряжения на каждом резисторе. Эти расчеты показаны ниже.

      ΔV 1 = I 1 • R 1 = (4 А) • (5 Ом)
      ΔV 1 = 20 В

      ΔV 2 = I 2 • R 2 = (2 А) • (8 Ом)

      ΔV 2 = 16 В

      ΔV 3 = I 3 • R 3 = (2 А) • (8 Ом)

      ΔV 3 = 16 В

      ΔV 4 = I 4 • R 4 = (4 А) • (6 Ом)

      ΔV 4 = 24 В

      На этом анализ завершен, и его результаты представлены на диаграмме ниже.

      Пример 2:

      Второй пример — более сложный случай — резисторы, включенные параллельно, имеют другое значение сопротивления. Цель анализа та же — определить ток и падение напряжения на каждом резисторе.

      Как обсуждалось выше, первым шагом является упрощение схемы путем замены двух параллельных резисторов одним резистором с эквивалентным сопротивлением.Эквивалентное сопротивление резистора 4 Ом и 12 Ом, включенного параллельно, можно определить, используя обычную формулу для эквивалентного сопротивления параллельных ветвей:

      1 / R экв = 1 / R 1 + 1 / R 2 + 1 / R 3

      1 / R экв = 1 / (4 Ом) + 1 / (12 Ом)

      1 / R экв. = 0,333 Ом -1

      R экв = 1 / (0,333 Ом -1 )

      R экв = 3.00 Ом

      На основании этого расчета можно сказать, что два резистора ответвления (R 2 и R 3 ) можно заменить одним резистором с сопротивлением 3 Ом. Этот резистор 3 Ом включен последовательно с R 1 и R 4 . Таким образом, общее сопротивление составляет

      . R до = R 1 + 3 Ом + R 4 = 5 Ом + 3 Ом + 8 Ом

      R до = 16 Ом

      Теперь уравнение закона Ома (ΔV = I • R) можно использовать для определения полного тока в цепи.При этом необходимо использовать общее сопротивление и общее напряжение (или напряжение батареи).

      I tot = ΔV tot / R tot = (24 В) / (16 Ом)

      I до = 1,5 А

      Расчет тока 1,5 А представляет собой ток в месте расположения батареи. При этом резисторы R 1 и R 4 включены последовательно, а ток в последовательно соединенных резисторах везде одинаков.Таким образом,

      I до = I 1 = I 4 = 1,5 А

      Для параллельных ветвей сумма тока в каждой отдельной ветви равна току вне ветвей. Таким образом, I 2 + I 3 должно равняться 1,5 А. Существует бесконечное множество значений I 2 и I 3 , которые удовлетворяют этому уравнению. В предыдущем примере два параллельно включенных резистора имели одинаковое сопротивление; таким образом, ток распределялся поровну между двумя ветвями.В этом примере неравный ток в двух резисторах усложняет анализ. Ветвь с наименьшим сопротивлением будет иметь наибольший ток. Для определения силы тока потребуется использовать уравнение закона Ома. Но для его использования сначала необходимо знать падение напряжения на ветвях. Таким образом, направление решения в этом примере будет немного отличаться от более простого случая, проиллюстрированного в предыдущем примере.

      Чтобы определить падение напряжения на параллельных ветвях, сначала необходимо определить падение напряжения на двух последовательно соединенных резисторах (R 1 и R 4 ).Уравнение закона Ома (ΔV = I • R) можно использовать для определения падения напряжения на каждом резисторе. Эти расчеты показаны ниже.

      ΔV 1 = I 1 • R 1 = (1,5 А) • (5 Ом)
      ΔV 1 = 7,5 В

      ΔV 4 = I 4 • R 4 = (1,5 А) • (8 Ом)

      ΔV 4 = 12 В

      Эта схема питается от источника 24 В.Таким образом, совокупное падение напряжения заряда, проходящего по контуру цепи, составляет 24 вольта. Будет падение 19,5 В (7,5 В + 12 В) в результате прохождения через два последовательно соединенных резистора (R 1 и R 4 ). Падение напряжения на ответвлениях должно составлять 4,5 В, чтобы компенсировать разницу между общим значением 24 В и падением 19,5 В на R 1 и R 4 . Таким образом,

      ΔV 2 = V 3 = 4,5 В

      Зная падение напряжения на параллельно соединенных резисторах (R 1 и R 4 ), можно использовать уравнение закона Ома (ΔV = I • R) для определения тока в двух ветвях.

      I 2 = ΔV 2 / R 2 = (4,5 В) / (4 Ом)
      I 2 = 1,125 А

      I 3 = ΔV 3 / R 3 = (4,5 В) / (12 Ом)

      I 3 = 0,375 А

      На этом анализ завершен, и его результаты представлены на диаграмме ниже.

      Разработка стратегии

      Два приведенных выше примера иллюстрируют эффективную концептуально-ориентированную стратегию анализа комбинированных схем.Подход требовал твердого понимания концепций последовательностей и параллелей, обсуждавшихся ранее. Такие анализы часто проводятся, чтобы решить физическую проблему для указанного неизвестного. В таких ситуациях неизвестное обычно меняется от проблемы к проблеме. В одной задаче значения резистора могут быть заданы, а ток во всех ветвях неизвестен. В другой задаче могут быть указаны ток в батарее и несколько значений резистора, и неизвестная величина становится сопротивлением одного из резисторов.Очевидно, что разные проблемные ситуации потребуют небольших изменений в подходах. Тем не менее, каждый подход к решению проблем будет использовать те же принципы, что и при подходе к двум приведенным выше примерам проблем.

      Начинающему студенту предлагаются следующие предложения по решению задач комбинированной схемы:

      • Если схематическая диаграмма не предоставлена, найдите время, чтобы построить ее. Используйте условные обозначения, такие как те, что показаны в примере выше.
      • При приближении к проблеме, связанной с комбинированной схемой, найдите время, чтобы организовать себя, записав известные значения и приравняв их к символу, например, I , I 1 , R 3 , ΔV 2 и т. Д. Схема организации, использованная в двух приведенных выше примерах, является эффективной отправной точкой.
      • Знать и использовать соответствующие формулы для эквивалентного сопротивления последовательно соединенных и параллельно соединенных резисторов. Использование неправильных формул гарантирует неудачу.
      • Преобразуйте комбинированную схему в строго последовательную, заменив (на ваш взгляд) параллельную секцию одним резистором, имеющим значение сопротивления, равное эквивалентному сопротивлению параллельной секции.
      • Используйте уравнение закона Ома (ΔV = I • R) часто и надлежащим образом. Большинство ответов будет определено с использованием этого уравнения. При его использовании важно подставлять в уравнение соответствующие значения. Например, при вычислении I 2 важно подставить в уравнение значения ΔV 2 и R 2 .

      Для дальнейшей практики анализа комбинированных схем рассмотрите возможность анализа проблем в разделе «Проверьте свое понимание» ниже.

      Мы хотели бы предложить … Зачем просто читать об этом и когда можно с этим взаимодействовать? Взаимодействовать — это именно то, что вы делаете, когда используете одно из интерактивных материалов The Physics Classroom. Мы хотели бы предложить вам совместить чтение этой страницы с использованием нашего интерактивного средства построения цепей постоянного тока.Вы можете найти его в разделе Physics Interactives на нашем сайте. Построитель цепей постоянного тока предоставляет учащемуся набор для построения виртуальных цепей. Вы можете легко перетащить источники напряжения, резисторы и провода на рабочее место, расположить и подключить их так, как вам нужно. Вольтметры и амперметры позволяют измерять ток и падение напряжения. Нажатие на резистор или источник напряжения позволяет изменять сопротивление или входное напряжение. Это просто. Это весело. И это безопасно (если вы не используете его в ванне).


      Проверьте свое понимание

      1. Комбинированная схема показана на схеме справа. Используйте диаграмму, чтобы ответить на следующие вопросы.

      а. Ток в точке A равен _____ (больше, равен, меньше) току в точке B.

      г. Ток в точке B равен _____ (больше, равен, меньше) ток в точке E.

      г. Ток в точке G равен _____ (больше, равен, меньше) ток в точке F.

      г. Ток в точке E равен _____ (больше, равен, меньше) току в точке G.

      e. Ток в точке B равен _____ (больше, равен, меньше) ток в точке F.

      ф. Ток в точке A равен _____ (больше, равен, меньше) ток в точке L.

      г. Ток в точке H равен _____ (больше, равен, меньше) ток в точке I.

      2. Рассмотрим комбинированную схему на схеме справа. Используйте диаграмму, чтобы ответить на следующие вопросы. (Предположим, что падение напряжения в самих проводах пренебрежимо мало.)

      а. Разность электрических потенциалов (падение напряжения) между точками B и C составляет _____ (больше, равно, меньше) разности электрических потенциалов (падение напряжения) между точками J и K.

      г. Разность электрических потенциалов (падение напряжения) между точками B и K составляет _____ (больше, равно, меньше) разности электрических потенциалов (падение напряжения) между точками D и I.

      г. Разность электрических потенциалов (падение напряжения) между точками E и F составляет _____ (больше, равно, меньше) разности электрических потенциалов (падение напряжения) между точками G и H.

      г. Разность электрических потенциалов (падение напряжения) между точками E и F составляет _____ (больше, равно, меньше) разности электрических потенциалов (падение напряжения) между точками D и I.

      e. Разность электрических потенциалов (падение напряжения) между точками J и K составляет _____ (больше, равно, меньше) разности электрических потенциалов (падение напряжения) между точками D и I.

      ф. Разность электрических потенциалов между точками L и A составляет _____ (больше, равно, меньше) разности электрических потенциалов (падение напряжения) между точками B и K.


      3.Используйте концепцию эквивалентного сопротивления, чтобы определить неизвестное сопротивление идентифицированного резистора, которое сделало бы схемы эквивалентными.




      4. Проанализируйте следующую схему и определите значения общего сопротивления, общего тока, а также тока и падения напряжения на каждом отдельном резисторе.


      5. Обращаясь к диаграмме в вопросе №4, определите …

      а. … номинальная мощность резистора 4.

      г. … скорость, с которой энергия потребляется резистором 3.

      Серия

      и параллельные резисторы

      • Изучив этот раздел, вы должны уметь:
      • Рассчитайте значения общего сопротивления в цепях с последовательным сопротивлением.
      • Используйте соответствующие формулы для расчета сопротивления в цепях с параллельным сопротивлением.
      • • Вычисление суммы обратных величин.
      • • Произведение над суммой.
      • Рассчитайте значения общего сопротивления в последовательной / параллельной сети.

      Расчеты в последовательно- и параллельных резисторных цепях

      Компоненты, включая резисторы в цепи, могут быть соединены вместе двумя способами:

      ПОСЛЕДОВАТЕЛЬНО, так что один и тот же ток течет через все компоненты, но на каждом из них может существовать разная разность потенциалов (напряжение).

      ПАРАЛЛЕЛЬНО, так что одинаковая разность потенциалов (напряжение) существует на всех компонентах, но каждый компонент может проводить разный ток.

      Рис. 4.2.1 Резисторы серии

      Рис. 4.2.2 Параллельные резисторы

      В любом случае (для резисторов) полное сопротивление той части цепи, которая содержит резисторы, может быть рассчитано с использованием методов, описанных ниже.

      Возможность рассчитать суммарное (общее) значение резисторов таким способом позволяет легко вычислить неизвестные значения сопротивления, тока и напряжения для довольно сложных схем с использованием относительно простых методов.Это очень полезно при поиске неисправностей.

      ПЕРЕД ДАЛЬНЕЙШЕЙ ДАЛЬНЕЙШЕЙ ПОПРАКТИКОЙ ИСПОЛЬЗУЙТЕ ФОРМУЛЫ ДЛЯ РАСЧЕТА ОБЩИХ ЗНАЧЕНИЙ СЕРИИ И ПАРАЛЛЕЛЬНЫХ РЕЗИСТОРОВ.

      Для резисторов в серии:

      Суммарное сопротивление двух или более резисторов, подключенных последовательно , определяется простым сложением индивидуальных значений резисторов, чтобы найти общую сумму (R TOT ):

      Для резисторов, включенных параллельно:

      Для расчета общего сопротивления цепи, в которой используются параллельные резисторы, можно использовать следующую формулу.

      Обратите внимание, однако, что эта формула НЕ дает вам общего сопротивления R TOT . Это дает вам ВЗАИМОДЕЙСТВИЕ R TOT или:

      Это совсем другое значение — и НЕ является полным сопротивлением. Он делится на 1, разделенный на TOT . Чтобы получить правильное значение для TOT рэндов (которое будет обратным 1/ TOT , т. Е. TOT /1, просто нажмите соответствующую клавишу на вашем калькуляторе (отмеченную 1 / x или x-1) .

      Другой способ расчета параллельных цепей.

      Суммарное сопротивление двух резисторов, включенных параллельно , без учета обратных величин, определяется по формуле:

      Эту формулу часто называют «произведение над суммой».

      Рассчитывает только ДВА резистора параллельно? Ну да, но это не большая проблема. Если имеется более двух параллельных резисторов, просто выберите два из них и определите общее сопротивление для этих двух — затем используйте это общее сопротивление, как если бы это был один резистор, и составьте еще одну пару с третьим резистором.Определите новую сумму и так далее, пока вы не включите все параллельные резисторы в этой конкретной сети.

      О, еще кое-что, что нужно помнить о произведении над суммой, видите скобки вокруг суммы (нижняя часть) формулы? Это означает, что вы должны решить это ДО того, как использовать его для разделения продукта (верхняя часть) на. Если вы этого не сделаете, ваш ответ будет неправильным.

      Звучит сложно? Не совсем, это просто вопрос повторения, и на практике вы не часто встречаетесь с множеством параллельных сетей с гораздо более чем двумя резисторами.Тем не менее, какую формулу вы выберете, зависит от вас, взаимная или сумма продукта.

      Подсказки

      Использование обратного метода

      Если вы используете МЕТОД ВЗАИМОДЕЙСТВИЯ для параллельных цепей, НЕ ЗАБУДЬТЕ, когда вы добавили обратные величины отдельных резисторов — вы должны снова найти обратную величину. 1 / R1 + 1 / R2 + 1 / R3 = 1 / R TOT и чтобы найти R TOT , вы должны найти обратное 1 / R TOT .

      Упрощающие схемы

      Для комбинированных последовательных и параллельных цепей сначала определите участок цепи (последовательный или параллельный).Затем перерисуйте схему, заменив участок, сопротивление которого вы нашли, одним резистором. Теперь у вас есть упрощенная схема, по которой можно найти R TOT .

      Вы можете использовать формулу «произведение на сумму»:

      Для цепей с более чем двумя параллельными резисторами просто определите два параллельных резистора одновременно, используя формулу произведения на сумму, а затем перерисуйте схему, заменив два резистора одним резистором, значение которого является объединенным сопротивлением двух .

      Теперь вы можете использовать первое комбинированное значение в качестве единственного резистора со следующим параллельным резистором и так далее. Таким образом, можно выработать большое количество параллельных резисторов с использованием произведения на сумму.

      Когда все параллельные резисторы одинакового номинала.

      Если несколько одинаковых параллельных резисторов подключены, общее сопротивление будет равно номиналу резистора, умноженному на обратную величину количества резисторов.

      , т. Е. Два параллельных резистора 12 кОм имеют общее сопротивление

      .

      12K x 1/2 = 6K

      Три параллельно включенных резистора 12 кОм имеют суммарное сопротивление

      12K x 1/3 = 4K и т. Д.

      Проверяю ответ

      Суммарное значение любого количества параллельных резисторов всегда будет МЕНЬШЕ, чем значение наименьшего отдельного резистора в сети. Используйте этот факт, чтобы проверить свои ответы.

      Серия

      и параллельная комбинация

      Попробуйте несколько вычислений на основе последовательной и параллельной цепей резисторов. Для этого вам просто нужно использовать информацию на этой странице и на странице «Советы по расчету резисторов». Вас просят вычислить общее сопротивление для каждой цепи.Вы можете выбрать, какую формулу использовать

      Вы также можете получить помощь по математике, загрузив нашу бесплатную брошюру «Советы по математике».

      Прежде чем начать, подумайте об этих нескольких советах. Они упростят задачу, если вы будете внимательно им следовать.

      1. Разработайте ответы с помощью карандаша и бумаги; перерисуйте схему, над которой работаете.

      2. Конечно, ответ — это не просто число, это будет определенное количество Ом, не забудьте указать правильную единицу (например.грамм. Ω, KΩ или MΩ) или ваш ответ не имеет смысла.

      3. Когда вы вводите значения в калькулятор, преобразуйте все значения KΩ или MΩ в Ом с помощью клавиши EXP. Если вы здесь ошибетесь, то получите действительно глупые ответы, в тысячи раз слишком большие или слишком маленькие.

      Итак, вы прочитали эти инструкции и готовы приступить к работе. Вот способ решить типичную проблему на бумаге, чтобы (со временем) вы не запутались.

      Пример последовательной и параллельной цепей

      .

      Хорошо, здесь есть что вспомнить, так почему бы не попробовать несколько практических вопросов в модуле резисторов 4.5 по определению общего сопротивления некоторых цепей резисторов?

      Формулы и калькулятор »Электроника

      Формулы, расчеты и калькулятор для определения общего сопротивления резисторов, установленных последовательно и параллельно.


      Учебное пособие по сопротивлению Включает:
      Что такое сопротивление Закон Ома Омические и неомические проводники Сопротивление лампы накаливания Удельное сопротивление Таблица удельного сопротивления для распространенных материалов Температурный коэффициент сопротивления Электрическая проводимость Последовательные и параллельные резисторы Таблица параллельных резисторов


      Резисторы могут быть размещены во многих конфигурациях в электрической или электронной схеме — иногда последовательно, иногда параллельно.

      Когда они размещаются в этих конфигурациях, важно иметь возможность рассчитать общее сопротивление. Этого можно довольно легко достичь, если использовать правильные формулы — есть простые формулы как для последовательных, так и для параллельных резисторов.

      При проектировании электронной схемы или по другой причине возможность вычисления сопротивления комбинации резисторов может быть очень полезной.

      В электронных схемах комбинации резисторов могут быть сведены к последовательным элементам и параллельным элементам, хотя при использовании других электронных компонентов комбинации могут быть более сложными.Однако во многих случаях расчет значений последовательного и параллельного сопротивления имеет большое значение.

      Резисторы серии

      Самая простая конфигурация электронной схемы — это резисторы, включенные последовательно. Это может произойти, если несколько этих электронных компонентов соединены последовательно, или необходимо добавить сопротивление кабеля к сопротивлению резистора и т. Д.

      Если резисторы соединены последовательно, то общее сопротивление является просто суммой отдельных резисторов.

      Последовательные резисторы

      Величину резисторов или сопротивлений, включенных последовательно, можно математически выразить следующим образом:

      Пример расчета последовательных резисторов:
      В качестве примера, если три резистора номиналами 1 кОм, 2 кОм и 3 кОм соединены последовательно, то общее сопротивление составит 1 + 2 + 3 кОм = 6 кОм.

      В реальных жизненных ситуациях и аспектах проектирования электрических и электронных схем будет много областей, где есть электронные компоненты, такие как резисторы или другие элементы, вносящие сопротивление, где необходимо суммировать ряд последовательно включенных сопротивлений.

      Сопротивления параллельно

      Есть также много случаев, когда электронные компоненты, такие как резисторы, а также другие элементы, вызывающие сопротивление, появляются в электрической или электронной цепи параллельно.

      Если резисторы размещены параллельно, они разделяют ток, и ситуацию немного сложнее вычислить, но все же довольно легко.

      1Rtotal = 1R1 + 1R2 + 1R3 + ……

      Пример расчета сопротивления резисторов, включенных параллельно:
      Чтобы дать пример, если есть три резистора, подключенных параллельно со значениями 1 кОм, 2 кОм и Омега и 3 кОм, то можно вычислить общее значение комбинации:

      1 / R Итого = 1/1000 + 1/2000 + 1/3000

      1 / R Итого = 1/1000 + 1/2000 + 1/3000

      1 / R Итого = 6/6000 + 3/6000 + 2/6000

      1 / R Итого = 11/6000

      R Всего = 6000/11 Ом или 545 Ом

      Корпус только двух резисторов, включенных параллельно

      Во многих конструкциях электронных схем наиболее распространенный экземпляр параллельных резисторов состоит только из двух электронных компонентов.

      Часто бывает так, что один резистор подключается параллельно другому. Или другой случай может быть, когда резистор помещается на клеммы для цепи или сети, которая имеет определенное сопротивление. В этом случае необходимо только рассчитать общее сопротивление для двух параллельно включенных резисторов.

      Если необходимо рассчитать общее значение для двух параллельных резисторов, уравнением можно манипулировать и значительно упростить его, как показано ниже:

      Эта формула значительно упрощает вычисление номинала двух параллельных резисторов, так как требует только одного умножения, одного сложения и одного деления.Часто это можно сделать мысленно или на клочке бумаги. В качестве альтернативы можно использовать наш простой калькулятор для двух параллельно включенных резисторов, приведенный ниже.

      Калькулятор для двух резисторов, включенных параллельно

      Этот калькулятор параллельного сопротивления обеспечивает простой метод расчета общего сопротивления для двух резисторов, соединенных параллельно.

      Хотя вычисление номиналов параллельных резисторов для двух резисторов упрощается до простой формулы, иногда гораздо проще и быстрее использовать калькулятор.

      Чтобы использовать калькулятор параллельных резисторов, просто введите значения параллельных резисторов в Ом, Ом или кОм и т. Д. В два поля ввода, но обратите внимание, что все значения должны быть в одних и тех же единицах, то есть оба в Ом кОм МОм и т. Д. Затем вычислитель параллельных резисторов предоставит общее сопротивление двух резисторов в тех же единицах, что и вход.

      Введите два значения для резисторов, R1 и R2, в поля, представленные в калькуляторе ниже, нажмите «Рассчитать», и будет предоставлено общее сопротивление.


      Калькулятор параллельного сопротивления

      Калькулятор параллельных резисторов позволяет легко рассчитать сопротивление двух резисторов, включенных параллельно, экономя записывать все и прибегая к ручке и бумаге или калькулятору в той или иной форме.

    Добавить комментарий

    Ваш адрес email не будет опубликован. Обязательные поля помечены *